Você está na página 1de 76

http://www.matgazine.

tk/

MATGAZINE REVISTA DE MATEMTICAS


Dirigida y coordinada por el consejo editorial:
Director Encargados de las secciones: Actualidad y novedades Entrevistas Artculos Problemas y soluciones Curiosidades y pasatiempos Vctor Arnaiz Adrin Toledano Moiss Herradn Gabriel Frstenheim Pablo Martn Pedro ngel Castillejo

Asociacin cultural

Lewis Carroll

Aula S-202, Facultad de Ciencias Matemticas, Universidad Complutense de Madrid (UCM) Ciudad Universitaria, Plaza de las ciencias 3, 28040, Madrid E-mail: matgazine@gmail.com URL:

www.matgazine.tk

Webmaster: Andrs Arcadio

http://www.matgazine.tk/

MATGAZINE MATHEMATICS MAGAZINE


Edited and coordinated by the editorial board
Director Managers of the sections: Currency and news Interviews Articles Problems and solutions Curiosities and puzzles Vctor Arnaiz Adrin Toledano Moiss Herradn Gabriel Frstenheim Pablo Martn Pedro ngel Castillejo

Cultural Association

Lewis Carroll

Adress: Aula S-202, Facultad de Ciencias Matemticas, Universidad Complutense de Madrid (UCM) Ciudad Universitaria, Plaza de las ciencias 3, 28040, Madrid E-mail: matgazine@gmail.com URL:

www.matgazine.tk

Webmaster: Andrs Arcadio

Editorial
Ya ha llegado el nuevo nmero de

Matgazine ! Tras la estupenda acogida

del nmero 0, decidimos en el equipo seguir adelante con la iniciativa y, tras nalizar los exmenes, pusimos en marcha este nmero. Lo primero de todo es agradecer a los estudiantes que han colaborado con la revista, ya sea escribiendo artculos o curiosidades. Mencin especial tienen aquellos que han resuelto e incluso propuesto problemas en la seccin de Problemas y soluciones. La estructura de la revista va a ser bsicamente la misma, aunque hemos abierto la posibilidad de que en la seccin de Artculos los profesores puedan escribir de forma voluntaria un artculo de divulgacin, como el que ha escrito para este nmero el profesor Gamboa, reas y longitudes. De esta forma los profesores que piensen que hay algn resultado curioso que no se suela ver en la carrera por falta de tiempo o porque no se ajusta al programa de ninguna asignatura tienen la opcin de escribir un artculo desarrollando el problema. En cada nmero habr, como mximo, un artculo escrito por profesores, para que la revista no pierda su esencia de ser un proyecto llevado por y para estudiantes. Otra novedad de la revista es su incorporacin a las redes sociales: bscanos en

Tuenti, en Facebook y en Twitter para estar al da de las ltimas

novedades. Asmismo hemos mejorado nuestra pgina web,

www.matgazine.tk,

en la que est disponible el anterior nmero de la revista en pdf. Por otro lado, la estructura interna del equipo ha quedado denida, crendose un consejo editorial que saca la revista adelante. En l estamos los encargados de cada seccin y el director, como se aprecia en la tabla de las primeras pginas. Tambin querramos agradecer todo tipo de donaciones que se efectuasen a la revista. Para ello, a partir del prximo nmero crearemos una seccin de agradecimientos para todas las personas o entidades que deseen colaborar econmicamente con el proyecto, pues debido a su bajo coste no cubrimos gastos, y no queramos subir el precio de la revista. Por ello, las personas o entidades que donis a partir de 5

a la revista apareceris en la seccin de

agradecimientos del prximo nmero. Por ltimo, nos gustara insistir en la peticin a los profesores para que participen en la seccin de Problemas y soluciones de la misma forma que lo han hecho los alumnos, mandando soluciones a los problemas ms difciles y proponiendo problemas originales que sean entretenidos. Muchas gracias! Y... a disfrutar!

El director

Actualidad y novedades
Olimpiada Matemtica Iberoamericana.
El pasado 6 de noviembre de 2010, se celebr en la Facultad de CC. Matemticas de la UCM la primera fase de la Olimpiada Matemtica Iberoamericana Universitaria en Madrid. La prueba constaba de un nico examen de siete problemas con una duracin de cinco horas, y se celebraba casi simultneamente en los distintos distritos universitarios espaoles y del resto de pases participantes. En cada pas, cualquier nmero de estudiantes puede participar, de manera no ocial, en la OIMU. Sin embargo, una vez efectuada la prueba se permite relacionar en el formulario de resultados que se enva al Coordinador Central, los detalles y resultados, ordenados por puntajes, de un mximo de 10 de estos estudiantes, quienes se convierten as en los representantes ociales del pas. Este ao, los 10 ganadores de la OIMU en Espaa son los siguientes. Enhorabuena a todos ellos

1.

Adrin Rodrigo Escudero

Nombre

Distincin
ORO PLATA PLATA BRONCE BRONCE BRONCE BRONCE M. HONOR

Moiss Herradn Cueto Gabriel Furstenheim Milerud


Ander Lamaison Vidarte Ivn Gener Fuenmayor David Alfaya Snchez Arnau Messegu Buisan

Francisco Criado Gallart Pedro ngel Castillejo Blasco


Jorge Alberto Jover Galtier

1 Los estudiantes destacados en negrita son alumnos de la Facultad de CC. Matemticas de la Universidad Complutense de Madrid.

Congresos y cursos de verano.

Escuela sobre Ecuaciones y Singularidades en Valladolid.


Los das 13 y 14 de junio de 2011, tendr lugar en la Universidad de Valladolid la cuarta edicin de la Escuela Doctoral ECSING Ecuaciones y Singularidades, que est abierta a estudiantes de todos los pases con atencin especial a los procedentes de la Unin Europea y de Iberoamrica. La Escuela programa cuatro cursos, de duracin semanal, completados por conferencias especializadas. Una modalidad de participacin es a travs de vdeo conferencia desde las universidades PUCP de Lima, Belo Horizonte y UNAM de Mxico. La fecha lmite para el envo de solicitudes es el 15 de mayo de 2011. Ms informacin en:

http://www3.uva.es/ecsing/
Centenario de la RSME. Congreso de Jvenes Investigadores en Soria.
Del 5 al 9 de septiembre de 2011 se celebrar en la Residencia Duques de Soria de esta ciudad castellana el Congreso de Jvenes Investigadores RSME. Esta actividad formar parte de las actividades que durante el ao 2011 conmemoran el Centenario de la Real Sociedad Matemtica Espaola (RSME). El objetivo del Congreso es compartir la reciente investigacin de los jvenes investigadores espaoles, propiciando el intercambio, la colaboracin y el conocimiento mutuo de los diversos trabajos realizados. El congreso tendr un amplio elenco de conferencias plenarias y varias sesiones paralelas (lgebra y Teora de Nmeros, Anlisis Matemtico, Geometra y Topologa, Matemtica Aplicada y Estadstica e Investigacin Operativa). Adems, habr una sesin de psters donde podrn presentarse trabajos o investigaciones en curso. Quienes deseen proponer un trabajo han de ponerse en contacto con el organizador de la sesin en la que se encuadre antes del 1 de junio de 2011 enviando un breve resumen. Ms informacin en

www.jirsme.uva.es

y en

www.rsme.es/centenario/
5

The Kentucky Section of the MAA invites you to MathFest 2011, Lexington, KY.
La MAA (Mathematical Association of America) te invita a ir a Lexington (Kentucky, USA) del 4 al 6 de agosto para el MAA MathFest 2011. La MAA planea un fantstico programa con variados invitados, numerosas sesiones de ponencias, debates, mini-cursos, actividades para estudiantes y mucho ms. La fecha lmite para apuntarse es el 17 de junio. Ms informacin en

http://www.maa.org/mathfest/mathfest.html
Congreso Maths & Earth en la Universidad de Zaragoza.
Del 15 al 17 de junio de 2011 se celebrar en Zaragoza el congreso Maths & Earth en la Universidad de Zaragoza. El objetivo de este evento, organizado por el Basque Center for Applied Mathematics (BCAM) y el Instituto de Matemticas y Aplicaciones (IUMA), es poner en contacto a cientcos de varios campos para analizar problemas actuales y cmo las matemticas pueden ayudar en la bsqueda de soluciones para los mismos. El plazo de registro naliza el 30 de mayo de 2011. Ms informacin en

http://iuma.unizar.es/maths_earth/
Logic Colloquium 2011. Barcelona, 11 al 16 de julio.
Del 11 al 16 de julio de 2011, se celebrar en Barcelona el Logic Colloquium. La ceremonia de apertura tendr lugar en el Paraninfo de la Universidad de Barcelona. Las conferencias plenarias y las sesiones especiales se realizarn en las aulas de la Facultad de Filosofa de la Universidad de Barcelona. El programa cientco constar de una conferencia sobre el trabajo de Gdel, a cargo de Anand Pillay (University of Leeds), dos tutoriales, por parte de Georges Gonthier (Microsoft Research), Formal Verication; y Martin Ziegler (Universitt Freiburg), Model Theory: Hrushovski's amalgamation constructions; y contar con conferenciantes plenarios como Steve Awodey (Carniege Mellon University), Lev D. Beklemishev (Steklov Mathematical Institute) y Jordi Lpez Abad (ICMAT Madrid), entre otros. Para el registro y la obtencin de toda la informacin:

http://www.logic2011.org/
XVI Cursos Universitarios de Verano en Lanzarote.
Durante el mes de julio de 2011 se celebrarn en Lanzarote los XVI Cursos Universitarios de Verano, organizados por la Academia de Ciencias

e Ingenieras de Lanzarote. El programa de dichos cursos contempla jornadas de Astronoma, cursos de Historia de la Matemtica, Cosmologa o Astrofsica. Ms informacin en:

http://www.academiadelanzarote.es /Cursos-de-Verano-Lanzarote-2011-Programa.htm
II Jornada SAGE/Python en Logroo.
Los das 28 y 29 de junio de 2011 se celebrar en Logroo la II Jornada SAGE/Python, organizada por la Universidad de La Rioja en colaboracin con el Instituto Universitario de Matemticas y Aplicaciones (IUMA) de la Universidad de Zaragoza y la Red EACA de Cculo Simblico, lgebra Computacional y Aplicaciones. El objetivo de esta jornada es compartir experiencias de tipo docente e investigador con SAGE/Python, mantener informados a los usuarios de las novedades que se vayan produciendo y animar a la comunidad matemtica a usar SAGE y colaborar en su desarrollo. El plazo de inscripcin naliza el 31 de mayo de 2011. Ms informacin en:

http://www.unirioja.es/dptos/dmc/sage2011/

Jornadas sobre los Problemas del Milenio, Barcelona, 1-3 de junio.

Estas jornadas son parte de las actividades que la Real Sociedad Matemtica Espaola (RSME) tiene programadas para el ao 2011, con motivo de su Centenario, y estn organizadas conjuntamente por la Facultad de Matemticas y el Instituto de Matemticas de la Universidad de Barcelona (IMUB). Los cursos estn dirgidos a estudiantes de segundo ciclo y doctorado, profesores en general, e investigadores en el rea. El programa constar de tres das completos, en los que cada maana y cada tarde se impartir un curso, empezando con un nivel bsico, apto para un pblico general, y aumentando la especializacin progresivamente. Cada curso corresponde a uno de los seis problemas del Instituto Clay que quedan pendienes: Las ecuaciones de Navier-Stokes, por Diego Crdoba (CSIC).

Existencia de Yang-Mills y de salto de mas, por scar Garca Prada (CSIC). La hiptesis de Riemann, por Ricardo Prez Marco (CNRS). La conjetura de Birch y Swinnerton-Dyer, por Vctor Rotger (Universidad Politcnica de Catalua). P versus NP, por Luis Miguel Pardo (Universidad de Cantabria). La conjetura de Hodge, por Vicente Muoz (Universidad Complutense de Madrid).

Los participantes recibirn un libro con las notas de los cursos, que estarn disponibles on-line un mes antes de la celebracin de las Jornadas. El IMUB ofrece 20 becas para estudiantes para cubrir la cuota de inscripcin. El nmero de plazas es limitado por lo que se recomienda la inscripcin anticipada. Ms informacin en:

http://garf.ub.es/Milenio/

Resuelta la conjetura de Nash por dos matemticos de la casa.

Ha sido una de las noticias de las matemticas espaolas ms difundida en los medios de comunicacin de los ltimos meses, junto con la resolucin del problema de los Conjuntos Generalizados de Sidon, por parte de Javier Cilleruelo, Carlos Vinuesa e Imre Ruzsa, y en Matgazine no podamos menos

que hacer tambin un poco de eco de ella, as como felicitar a los dos matemticos protagonistas.

Javier Fernndez de Bobadilla, granadino de 38 aos, licenciado en Mara Pe Pereira, burgalesa de 30 aos, tambin licenciada por la UCM, becaria
matemticas por la UCM y Cientco Titular del ICMAT (CSIC) y postdoctoral de Cajamadrid en el Instituto Jussieu de Pars, y que est a punto de defender su tesis en la Universidad Complutense de Madrid, han resuelto una conjetura que Nash plante en 1968. El problema de geometra algebraica se encuadra en el contexto de intentar resolver las singularidades en caracterstica cero. Segn dice el propio Javier, en un pequeo artculo para el blog en el que an trabaja.

http://gaussianos.com

, lleg al pro-

blema motivado por otro problema distinto de equisingularidad topolgica,

Tuvo un primer resultado que permiti introducir mtodos topolgicos en el problema. Mara empez a trabajar en el contexto de su tesis doctoral (en la Universidad Complutense de Madrid), en la que resolvi el caso de supercies cociente, e introdujo muchas ideas clave para la solucin nal. Posteriormente, pudieron dar una demostracin del caso general entre los dos. El 28 de abril, Javier dio una conferencia en la Facultad de Matemticas de la UCM, dentro del Seminario de Geometra Algebraica, en la que desarroll las ideas de su demostracin ante la atenta atencin de algunos de los que fueran sus profesores en la facultad. Como dijo el propio Javier, este era el sitio donde ms me apeteca dar esta charla.

John W. Milnor, premio Abel 2011.

El profesor John W. Milnor, del Instituto de Ciencias Matemticas de la Universidad de Stony Brook (Nueva York) ha sido galardonado con el premio Abel 2011, otorgado por la Academia Noruega de Ciencias y Letras. El jurado del premio Abel ha destacado sus descubriemientos pioneros en topologa, geometra y lgebra. El profesor Milnor naci en 1931 en Nueva Jersey, y desde 1953 (antes incluso de nalizar su tesis) hasta 1967 ocup distintas plazas de profesor en la Universidad de Princeton. En 1970, pas a formar parte del Instituto de Estudios Avanzados de Princeton. En 1989 se convirti en el primer director del Instituto de Ciencias Matemticas de la Universidad de Stony Brook, donde contina en la actualidad. El trabajo de John Milnor ha causado una profunda huella en las matemticas de los ltimos 60 aos. Prueba de ello son los mltiples objetos matemticos que reciben su nombre, como las esferas de Milnor, la bracin de Milnor, el nmero de Milnor y sus conjeturas en teora de nudos, teora K, teora de grupos combinatoria y sistemas dinmicos holomorfos. Es autor de numerosos libros y artculos de gran inuencia. Su monografa sobre singularidades aisladas en hipersupercies es considerada el trabajo ms importante de la teora de singularidades. En este nmero analizaremos en una de las curiosidades una de sus aportaciones ms curiosas, las llamadas esferas exticas.

10

Concurso de Primavera de Matemticas 2011.

El sbado 9 de abril, la Facultad de Matemticas acogi un ao ms el Concurso de Primavera, este ao en su XV edicin. 3136 estudiantes de entre 10 y 18 aos, que estudian en 483 centros educativos de la Comunidad de Madrid acudieron puntuales a la cita en la fase nal. Aproximdamente 37000 alumnos ya haban participado previamente en la primera fase en sus centros, y los nalistas, divididos en cuatro grupos (ltimo ciclo de Primaria, primer ciclo de Secundaria, segundo ciclo de Secundaria y Bachillerato) se enfrentaron a hora y media ininterrumpida ante los problemas propuestos. El entusiasmo, tanto de los participantes, como de sus padres y profesores acompaantes se palpaba en toda la Facultad, ms llena de gente que en cualquier otro da del ao. Los profesores, con Mara Gaspar, Joaqun Hernndez y Merche Snchez a la cabeza, conserjes y los numerosos estudiantes de la Facultad que acudieron voluntarios, se afanaron en que todo discurriera perfectamente durante toda la maana. El acto de entrega de premios se celebr el da 13 de abril, y a l asistieron el rector Berzosa; el decano de matemticas, Javier Montero; el director general de Mejora de la Calidad de la enseanza de la Comunidad de Madrid, Xavier Gisbert, y el profesor Joaqun Hernndez, presidente de la Asociacin Concurso de Primavera. En sus discursos, las autoridades felicitaron a los alumnos por su afn de mejora y esfuerzo, a sus familiares por el apoyo y la preocupacin por la formacin de los estudiantes, a sus profesores por su constante estmulo, y a todos los que hacen posible el desarrollo del concurso. Desde Matgazine, animamos a los profesores, y sobre todo a los estudiantes de la facultad, a participar para que este evento y otros muchos sigan celebrndose en favor de la divulgacin y la promocin de las matemticas en la sociedad.

11

ENCUENTRA MATEMTICAS 2011

de Madrid Pensamiento Matemtico

organiza la COMPETICIN ENCUENTRA MATEMTICAS 2011: GYMKHANA MATEMTICA.

El Grupo de Innovacin Educativa (GIE) de la Universidad Politcnica

El concurso est dirigido a alumnos de grado y postgrado matriculados en cualquier universidad espaola. Consiste en una Gymkhana que se realizar en equipos de dos a cuatro estudiantes, durante el da 30 de Septiembre de 2011, teniendo como escenario el Campus Moncloa de la Ciudad Universitaria de Madrid. Se citar a los grupos participantes a las 9,30 de la maana en el aula de seminarios de la Escuela de Caminos de la Universidad Politcnica (Avda. Profesor Aranguren s/n. 28040 Madrid) donde se les har entrega de la documentacin necesaria. La Gymkhana consistir en la resolucin de una serie de problemas que se propondrn a los grupos. Dichos problemas de contenido matemtico estarn relacionados con diversos aspectos de las Facultades, Escuelas, zonas comunes,...del campus, por las que deben pasar los grupos en su recorrido por la Ciudad Universitaria. Los problemas, una vez resueltos por los grupos participantes, se entregarn en los puestos de control que se situarn en los puntos que componen el recorrido. Los grupos participantes debern resolver adems otros problemas propuestos durante el recorrido (problemas de trayecto). Una vez nalizado ste, los grupos se volvern a dirigir a la Escuela de Caminos, jndose una hora tope para su llegada que se le indicar a cada grupo en su documentacin. En el puesto de control de dicha Escuela los grupos debern entregar sus soluciones a los problemas de trayecto. Los problemas entregados por cada grupo sern corregidos por un tribunal durante la tarde del da de competicin. Se valorar la idoneidad de los mtodos utilizados en la resolucin, la originalidad de los mismos y los resultados obtenidos, lo que dar una puntuacin en cada problema sobre un mximo que se determinar en el enunciado del mismo. La correccin se llevar a cabo por un tribunal
12

compuesto por profesores seleccionados por el comit organizador. Las decisiones de dicho tribunal sern inapelables. Se valorar con puntos extra el tiempo de realizacin de cada problema. Para ello se llevar control de los instantes en los que cada equipo recoge y entrega cada uno de los problemas en los puntos de control. Se clasicar a los grupos segn la puntuacin total obtenida. Los premios nales de la competicin sern de 1300 euros para el primer equipo y 800 euros para el segundo. Para la inscripcin y obtencin de ms informacin: http://www.caminos.upm.es/matematicas/concursoem2011/premios.html

XIV Spanish Meeting on Computational Geometry


Del 27 al 30 de junio, se celebra en la Universidad de Alcal, en Alcal de Henares, Madrid, la XIV edicin de los Encuentros de Geometra Computacional. En esta ocasin, la edicin tendr un especial carcter internacional en ocasin del 60 aniversario de Ferrn Hurtado. El programa consta de numerosas exposiciones breves de trabajos en geometra discreta y combinatoria, programacin lineal aplicada a problemas geomtricos, algoritmos geomtricos en estructuras de datos, fundamentos tericos de la geometra computacional, cuestiones de inters en la implementacin de algoritmos geomtricos y aplicaciones de la geometra computacional (grcos, realidad virtual, robtica, simulacin y visualizacin, modelizacin del slido, CAD, procesamiento de imgenes, multimedia y animacin, etc). Entre los conferenciantes invitados estn Manuel Abellanas, Oswin Aichholzer, Jin Akiyama, Jit Bose, Erik Demaine, Alberto Mrquez, Joe Mitchell, Jnos Pach, Vera Sacristn, Godfried Toussaint y Jorge Urrutia. Como curiosidad, hay que decir que Jin Akiyama es posiblemente el matemtico ms conocido de la actualidad en el mundo, y probablemente de la historia. La razn es que Jin Akiyama tiene un programa de televisin en horario de mxima audiencia en la NHK japonesa desde el ao 1991!. Adems de su programa de televisin, Jin ha aparecido en pelculas, series de televisin, en videojuegos de Nintendo, ha sido personaje de algn manga, presenta programas de radio, hasta tiene su propio museo. Para ms informacin sobre el Encuentro:

http://www2.uah.es/egc2011/inicio.html

13

Entrevista
Entrevista a la profesora Begoa Vitoriano
Begoa Vitoriano es profesora titular del departamento de Estadstica e Investigacin Operativa de la Facultad de CC. Matemticas de la UCM. La profesora Vitoriano y su grupo de investigacin compaginan las matemticas con la labor humanitaria, resolviendo problemas logsticos y colaborando con universidades de pases en vas de desarrollo.

Matgazine:

Qu te impuls a estudiar matemticas?

Begoa Vitoriano: Es una pregunta difcil, en realidad yo quera estudiar


periodismo. Me gustaba escribir y viajar; sin embargo, a medida que pasaba el tiempo iban llamndome ms la atencin las matemticas y la astronoma. No fue una decisin fcil; cuando estaba estudiando COU mi padre falleci y mi madre quera que trabajase para ayudar a la familia. Ella prefera que estudiase una carrera ms normal, como Derecho. Hace unos aos mucha gente tena la idea de que matemticas no era una carrera con porvenir. Pero siempre he sido un poco rebelde y empec la carrera costendomela a base de clases particulares.

MZ: Hay alguna gura que te haya inspirado especialmente como cientca y persona? BV: Qu difcil! En el colegio siempre me llam la atencin Marie Curie
como gran cientca. Sin embargo, quienes ms me han inuido en mi carrera estn ms cerca. Profesionalmente, han sido Javier Montero y Juan Tejada, que en su da fueron profesores mos y con quienes he tenido el placer de trabajar. Tambin, Andrs Ramos, un profesor de la E.T.S. de Ingeniera (ICAI) con el que trabaj. Como compaera, claramente Mara Teresa Ortuo, tambin profesora de esta facultad. Aprecio su forma de pensar y que en algn momento me haya ayudado a poner los pies en el suelo.

MZ:

Por qu decidiste especializarte en Investigacin Operativa?

BV: Lo decid casi desde el principio, en primero. Con el plan de estudios


que curs haba que elegir alguna especialidad en tercero, entonces daban charlas a los estudiantes de segundo para orientarles. Yo me col en una de las charlas de investigacin operativa. Me gust mucho lo que decan, la utilidad que poda darle a las matemticas. En esa poca pensaba en ser agente de bolsa, orientar mi carrera a las nanzas y a ganar dinero. As que la idea inicial de la astronoma, que me atrajo a la carrera, qued un poco en el olvido. La

14

otra especialidad que ms me llamaba la atencin era fundamental, el anlisis me encantaba pero la geometra hizo que lo descartase denitivamente.

MZ:

Cmo recuerdas tu poca universitaria?

BV: Feliz, muy feliz. Durante esos aos tena casi todo el tiempo ocupado,
con clases por la maana y trabajo por la tarde. Por aquel entonces jugaba al rugby a nivel nacional, le dedicbamos bastante tiempo y nuestro equipo gan el campeonato de Espaa varias veces. En el tercer ao de carrera tuve a mi primer hijo y me independic, segua viviendo a base de dar clases particulares. Ms adelante, en quinto obtuve una beca de Hewlett-Packard. S, sin duda fueron unos aos muy ocupados, y muy felices.

MZ:

Qu son para ti las matemticas?

BV: Todo. Son mi vida, me dedico a ensearlas, estudiarlas e investigarlas.


Son mi trabajo, y tengo la suerte de que me encantan. Qu ms puedo pedir?

MZ: Podemos decir que el proyecto humanitario que ests llevando a cabo se divide en dos ramas, por un lado el desarrollo de herramientas matemticas para su utilizacion en operaciones humanitarias en desastres, y por otro el de cooperacin al desarrollo. Explcanos en qu consisten exactamente estas actividades. BV: Un alumno, Antonio Omaa, de ingeniera industrial en la Universidad de Comillas, donde era profesora, me propuso que le dirigiese el trabajo de mster. l quera profundizar en el estudio de la logstica humanitaria. Le ped que no fuese algo vaco, que encontrase algo donde aplicar el trabajo a desarrollar. Despus de un tiempo buscando, se pusieron en contacto con nosotros Cruz Roja Espaola y la OMS de Panam. Decidimos colaborar con Cruz Roja Espaola. As empezamos a trabajar en SEDD (Sistema Experto de Diagnostico de Desastres). SEDD es una herramienta que sirve para ayudar a las ONG's en la toma de decisiones ante catstrofes humanitarias. Se parte de unos datos objetivos obtenidos en el terreno, la intensidad en la escala Richter de un terremoto, por ejemplo, y se utiliza una base de datos histrica de la que obtenemos los datos de lo ocurrido en situaciones similares. Hacemos una clasicacin con el n de tener en cuenta lo que no ha ocurrido con anterioridad. Empleando lgica difusa podemos clasicar la gravedad de la catstrofe para ayudar a decidir la clase de despliegue que ha de hacer la organizacin. Hasta ahora, esta decisin corresponda a un miembro de Cruz Roja con mucha experiencia. Esta herramienta plantea ser un apoyo, y no un sustituto, de esa persona. Desarrollamos tambin HADS, que de forma simplicada se trata de un modelo de ayuda a la decisin para las organizaciones en el problema de distribucin

15

de ayuda disponible en el terreno. La diferencia con la logstica comercial es que mientras all los factores a tener en cuenta suelen ser econmicos, para la logstica humanitaria es mas importante el hecho de ser capaces de mantener comunicados todos los nodos, entre otras cosas por el criterio de humanidad o equidad que impera en estas operaciones. Hay que tener en cuenta que en una zona de conicto o catstrofe natural, los enlaces entre los puntos de distribucin: aeropuertos, puertos, etc. y los nodos de recogida pueden romperse por diversas razones. La crecida excesiva de un ro puede dejar inoperativo un puente. Tambin se trata de cuidar la seguridad de un convoy evitando zonas de riesgo por asalto, etc.

MZ:

Y el proyecto de cooperacin al desarrollo?

BV: En 1995 viaj a El Salvador, casi por casualidad. El recuerdo que


tengo del primer contacto es impresionante. All todo es muy clido, la gente te acoge muy bien, la hospitalidad es enorme. Sin embargo, hubo momentos muy difciles. En esa poca el pas se encontraba en una situacin verdaderamente dura, saliendo haca no mucho de una terrible guerra civil. Al ao siguiente comenc a coordinar el programa de mster de estadstica. Ms tarde tambin llevamos un mster de estadstica a Per. Ms adelante, fui a Mozambique con una ONG para identicar dnde podamos colaborar: formacin de lderes, uso de ordenadores en zonas con electricidad, donaciones, etc. Pero en un momento dado, me llev unos alumnos de ltimos cursos de ingeniera industrial, y a mis hijos, y me vi enseando Excel con ellos. Ah me di cuenta de que algo estaba mal, porque si ellos pueden hacerlo no tiene mucho sentido que lo haga yo. No tiene sentido porque yo puedo hacer otras cosas que ellos no pueden: formar a los profesores de la universidad. Entonces retom la cooperacin universitaria y me puse en contacto con las universidades porque le daba ms sentido a la cooperacin que yo poda hacer. A pesar de todo, durante varios aos he mantenido en paralelo la parte de cooperacin universitaria y la de identicacin de proyectos de desarrollo rural. Ahora tenemos en Mozambique un curso de postgrado que pretendemos que el ao que viene se convierta en mster como el de El Salvador. Ha ido todo muy despacio, primero me puse en contacto con la Universidad Pedaggica y di un curso, luego dentro del mster que ellos tenan di otro curso de Estadstica en Educacin. Ms adelante pusimos un curso de especialista en estadstica para ver si entendan sucientemente bien castellano y vimos que el idioma no era un problema. Este ao se est repitiendo, aunque hubiese preferido que ya fuera un mster, y queremos por n el ao que viene conseguirlo. All ya se est creando con todo profesorado espaol. Es una inversin muy fuerte, porque los profesores que van no tienen un reconocimiento en su carga

16

laboral aqu de ningn tipo. Por eso confo en la gente joven que suele tener ms disponibilidad porque a pesar de que tienen bastantes cosas que hacer no tienen cargos que es algo que te ata mucho y te impide marcharte ms de un par de semanas.

MZ:

Y volviendo a Madrid, qu mejoraras de la facultad de matemticas?

BV: Muchas cosas... hay muchos palos muy distintos. Probablemente, en


investigacin habra que buscar una forma para dar a conocer las matemticas fuera de aqu. Los investigadores somos todos muy buenos, nos conocemos entre nosotros pero al nal no nos conoce nadie ms. Hay mucha gente aplicando matemticas que vienen de otros mundos, como ingenieras, etc. porque no se sabe que el matemtico tiene muchas capacidades. Es verdad que nos colocamos muy bien y la carrera est valorada, pero no por un pblico general. Por eso creo que debera haber un esfuerzo en los departamentos por hacer una transferencia a la sociedad con proyectos y dems. No hablo slo de empresas, pienso en ONGs y otras entidades. Hay muchas cosas que se pueden transferir y no se hace. No se hace, supongo, porque suponen un esfuerzo que no est valorado. Eso por una parte; por otra, me parece que falta inquietud por parte del alumnado por el asociacionismo y por la colaboracin. No veo que la gente se mueva para preguntar, para informarse sobre los proyectos, etc. A lo mejor me equivoco, pero me da la impresin de que la facultad est ms parada que hace unos aos. Me sobran tambin algunos alumnos demasiado obsesionados por la gran nota. Es un espritu muy competitivo que siempre he odiado y que no lo vea cuando estudi; creo que ha ido creciendo y creciendo hasta parecerme vergonzoso. Creo que los profesores no deberamos fomentar cosas de este estilo. Cmo cambiarlo es algo que me parece ms complicado. Me gustara que la autonoma del alumno tuviese ms peso. En este sentido, totalmente en contra de Bolonia. Creo que no tiene sentido que el alumno que falta a clase un da ande justicndose, esto es la universidad. Hay una parte, quiz la ms importante, que es el estudio autnomo. No puede ser que un alumno est tan preocupado por la clase a la que ha faltado; a m no me convence. Pienso que hay muchas formas de aplicar la evaluacin continua y que no consisten en un examen semanal. Pero sobre todo habra que dejar una cierta autonoma. Luego cambiara ciertas cosas en general de la universidad, que fue diseada hace tiempo y quiz haya gente que se pueda aprovechar de ello. Viendo a los profesores y su dedicacin por ejemplo. Me gustara que no hubiera nadie que se pudiese rascar la barriga la mitad de la jornada mientras otros estn desbordados. Pero eso son problemas generales de la educacin, de cmo medir la actividad del profesorado y cmo valorarla.

17

MZ:

Y con qu te quedas de ella?

BV: Con casi todo. Renunci a una plaza ja en un buen puesto en una
universidad privada por venir aqu, porque creo en la universidad pblica, en la educacin pblica en general. Estoy en contra de la privatizacin de la educacin. Me parece un servicio tan bsico que no creo que se deba privatizar, porque todo lo que se hace en ella debe estar controlado. A parte, en concreto me quedo con la facultad de matemticas de la Complutense porque es de las pocas facultades de matemticas que hay en Espaa con un nmero suciente de gente, muy buena, en todas las reas. Con lo cual tiene una gran capacidad para la especializacin y para que la investigacin sea diversa. Adems, creo que el grupo de investigacin operativa de esta facultad es el mejor y ms especializado de Espaa. En general, tener tres grados, poder especializarte tanto, es lo que marca la diferencia con el resto de facultades de matemticas. Creo que el plan del 95 es el peor que ha habido por ese mismo motivo. El que yo curs era bueno porque te permita formarte muy bien en tu rea. El del 95 ha conseguido que no tengis perl y andis un poco perdidos, creo que el que viene mejora eso. Todo esto lo tiene esta facultad y no otras. Me gusta mucho la facultad por su tamao y la posibilidad que brinda formar diferentes perles de matemticos. Me parece un buen sitio para estudiar y trabajar.

MZ: Qu les diras a los estudiantes que todava no han cursado probabilidad, estadstica, etc.? BV: Bueno, qu ms decir despus de lo que te he contado sobre logstica
humanitaria y la aplicacin de la matemtica. Creo que EIO es lo que pasa a la sociedad de forma ms inmediata. Yo imparto Probabilidad por gusto, me gustaba el anlisis, la teora de la medida, etc. Y la investigacin operativa es lo que ms me gusta y es a lo que me dedico, en lo que investigo y trato de transferir a la sociedad. Es bonito intentar resolver problemas reales. En ese sentido se parece a la fsica. Pero la fsica se centra ms en la naturaleza, etc. A m me gusta ms donde hay personas, como en los problemas de gestin. Es la transferencia directa de las matemticas a la realidad. La parte de la estadstica me gusta, aunque quiz no tanto las matemticas que lleva detrs. Sin embargo, la parte de prediccin o clasicacin es muy interesante. Por ejemplo, el problema de SEDD es un problema de clasicacin que tradicionalmente ha sido de un problema de estadstica, pero no emplea procedimientos de estadstica clsica. Como idea el intentar predecir qu ocurrir en el futuro o explicar qu ha ocurrido es muy divertido. Animo a estudiar estadstica, aunque es cierto que las matemticas que lleva, desconectadas del problema, pueden no gustar a todos. De todos modos, lo que es cierto es que

18

la aplicacin al mundo real es inmediata. Otras ramas son muy bonitas, pero no ves tan fcilmente el problema real al que lo aplicas.

MZ:

Algn consejo para los estudiantes, y lectores en general?

BV: Bueno, yo me dedico a los sistemas de ayuda a la decisin as que mi


consejo es que cuando tomen sus decisiones intenten decidir despus de pensar y estudiar los pros y contras, segn lo que les mande su vocacin. Creo que decidir hacer una cosa porque es lo que nos conviene para el futuro, aunque no nos guste nada, es algo que crea personas tristes. A la hora de decidir en general, est bien no hacer las cosas a lo loco, pero hay que dejarse llevar por la intuicin y mantener la ilusin.

19

Artculos
El conjunto de Cantor
Los conceptos de dimensin y medida vigentes hasta el s. XIX se vieron cuestionados por la aparicin de ciertos conjuntos con propiedades singulares, posteriormente denominados fractales. El conjunto de Cantor es un ejemplo de este tipo de conjuntos. A lo largo de este artculo comprobaremos este hecho con el conjunto de Cantor, mostraremos unas denicines de dimensin y medida que solucionan los problemas planteados por este tipo de conjuntos y veremos ejemplos de variantes del conjunto de Cantor.

Cantor desafa las dimensiones enteras


Vamos a comenzar con la construccin/descripcin del conjunto de Cantor,

C0 el intervalo [0,1] de la recta real. Llamamos C1 al resultado de C0 el intervalo abierto central de longitud 1/3, es decir, el intervalo 1 2 , 3 . Para obtener C2 repetimos el mismo procedimiento con C1 , esto es, 3 7 8 1 2 suprimimos los intervalos 9 , 9 y 9 , 9 . Iterando el proceso, n N obtenen mos el conjunto Cn que est formado por 2 intervalos cerrados de longitud n 3 . Consideramos C = n0 Cn , el conjunto de Cantor.
Sea quitar a

C.

Figura 1: Seis primeras iteraciones de la construccin del conjunto de Cantor.

Como

Cn [0, 1],

parece natural que como un segmento tiene dimensin

1, el conjunto C tambin tenga dimensin 1, al ser

n N.

Tenemos que

Cn

es la unin de

su longitud (medida unidimensional)

Cn unin de intervalos 2n intervalos de longitud 3n , luego n n es 2 3 n N. Segn n crece al ir

20

iterando, tenemos que la longitud tiende a 0, al ser

C Cn n N,

deducimos que

2 3 < 1. Luego, como tiene medida nula. Intuitivamente nos hace

cuestionar si es procedente asignarle dimensin 1. Una propiedad de tervalo

es que est formado por todos los nmeros del in-

[0, 1]

que admiten una expresin en base 3 que solo contiene ceros y

doses, ya que en cada paso se suprime el intervalo central. Si cambiamos los doses por unos obtenemos todos los nmeros del intervalo

[0, 1]

que admiten

una expresin con unos y ceros, o lo que es lo mismo, todo el intervalo no puede ser menor que el del intervalo lo tanto, no parece razonable asignar a encontrar una medida apropiada para

expresado en base 2. Esta aplicacin es sobreyectiva, luego el cardinal de

[0, 1] C

[0,1], luego C es no C dimensin nula.

numerable. Por

Luego tenemos la necesidad de introducir dimensiones fraccionarias para

C.

Medida y dimensin de Hausdor


El matemtico alemn Felix Hausdor deni un tipo de medida que solventaba los problemas que planteaban los conjuntos fractales como el conjunto de Cantor. Asociada a esa medida tambin deni una dimensin. Vamos a denirlas y a estudiar alguna de sus propiedades. Para denir la medida de Hausdor, necesitamos los conceptos de dimetro de un conjunto y de

recubrimiento,

siendo

se denota |E|, como sup{||x y|| : x, y E}, donde || || es la norma eucldea. Se dice que {Ui : i I N} es un recubrimiento de un conjunto E si E iI Ui y |Ui | < i I . Se dene el dimetro de un conjunto

> 0. E Rn , y

Denicin. Dado
de Hausdor,H
s

E Rn , jado s > 0 se (E), de la siguiente manera


0

dene la medida

s-dimensional

H s (E) = l H (E) m s
siendo
s H (E) = nf{ iI

|Ui |s : siendo {Ui } un recubrimiento de E}

y siendo

I N.

Teorema 1. La medida de Hausdor es una medida exterior.


Demostracin. Primero vamos a comprobar que
es decir, cumple las siguientes propiedades:

s H

es una medida exterior,

21

s H () = 0 s s II) Si A B encontes H (A) H (B) s s III) H ( i=1 Ai ) i=1 H (Ai ).


I)

U = {0} un recubrimiento de s s |U | = 0, tenemos que 0 H (E) 0, luego H () = 0. II) Sea {Ui : i I} un -recubrimiento de B, entonces {Ui : i I} es un -recubrimiento de A, por lo que
I) Tomando como recubrimiento del vaco Como

{
iI

|Ui |s : {Ui } recubrimiento de B i I} |Ui |s : {Ui } recubrimiento de A i I}


iI

{
Tomando nmos,

s H (B) = nf{ iI

|Ui |s : {Ui } recubrimiento de B i I}

nf{
i=1

s |Ui |s : {Ui } recubrimiento de A i I} = H (A)

i=1 Ai lo ordenamos de la siguiente manera: primero los que intersecan con A1 , luego los restantes que intersecan con
III) Dado un recubrimiento de

A2
con

y as sucesivamente. Utilizamos la siguiente notacin de acuerdo con esta

ordenacin:

es el conjunto de los ndices de los conjuntos que intersecan

Aj j < i, i N; es el conjunto de los ndices i de los conjuntos que intersecan con Ai . Obviamente, i i Destaquemos s s que H (Ai ) = nf{ j |Uj | } ya que los conjuntos que no intersecan con i Ai pueden suprimirse para calcular el nmo. Ai
y no intersecan con Ahora,

s H i=1

Ai

= nf{
iI

|Ui |s : {Ui } recubrimiento de


i=1

Ai } =

= nf{
i=1 ji

|Uj |s } =
i=1

nf{
ji s H (Ai ) i=1

|Uj |s }

i=1

nf{
j i

|Uj |s } =

22

ya que, como hemos dicho, podemos suponer que todos los conjuntos de un recubrimiento de

Ai

intersecan con l.

Hemos comprobado que

s H

es medida exterior

buen comportamiento de los lmites con las desigualdades

> 0, luego por el , deducimos que

l H = H s m s

es tambin una medida exterior.

Teniendo ya denida la medida de Hausdor, vamos a comprobar que existe un valor de

para el cual la medida de un conjunto

E Rn

tiene su

verdadero sentido. Adems, vamos a ver que ese valor es nico. Como la medida de Hausdor se obtiene tomando el lmite cuando podemos suponer que

< 1.

Sea entonces

E Rn , I

un conjunto de ndices, y

0, {Ai : i I} un -recubrimiento de sean t, s R con t > s. Tenemos que |Ai |s


iI

|Ai |t =
iI iI
y tomando nmos en todos los

|Ai |ts |Ai |s ts -recubrimientos

y aplicando la denicin de

medida de Hausdor, tenemos que

t s H (E) ts H (E)
Ahora, cuando

0 observamos que si H s (E) < , entonces H t (E) = 0, t ts mientras que si H (E) > 0 entonces como 0, H s (E) = +. t Luego existe un nico valor de s tal que si t < s, H (E) = + y si t > s t entonces H (E) = 0. Se dene la dimensin de Hausdor, y la denotamos dimH (E), como dicho valor de s. Si el conjunto E es nito, entonces H 0 (E) < , E tiene dimensin de Hausdor igual a 0. Si E es innito, tenemos que H 0 (E) = y el sup{s : H s (E) = } es un nmero mayor o igual a cero, que coincide con la dimensin de Hausdor del conjunto E . Adems, se puede
probar que

0 dimH (E) = sup{s : H s (E) = } = nf{s : H s (E) = 0} n


En el conjunto de Cantor observamos que contiene copias de l a escala

1 3 C . Para ello vamos a estudiar el caso general en el que se reduce a escala k (0, 1) el n conjunto E R , es decir, aplicar a E una homotecia de razn k . Sea {Ai : i I} un -recubrimiento de E , I un conjunto numerable de ndices. Tenemos que
1/3, luego es interesante ver qu relacin hay entre

H s (C)

Hs

si aplicamos la reduccin a escala

{kAi : i I}

es un

k al conjunto E y a los conjuntos Ai i I , k -recubrimiento de kE . Ahora,


|kAi |s = k s
i=1 i=1

|Ai |s

23

Tomando nmos sobre todos los recubrimientos de

E:

s s Hk (E) = k s H (E)
y tomando el lmite cuando

0,

obtenemos la relacin:

H s (kE) = k s H s (E)
Adems, si el conjunto mismo a escala

es autosemejante con

n N

copias iguales de s

k,

tenemos que

1 s H (E) = k s H s (E) n
luego obtenemos la siguiente frmula para la dimensin

s=

1 log n log k

Dimensin y medida del conjunto de Cantor


Vamos a calcular ahora de manera informal cul es la candidata a ser la dimensin de Hausdor de ser su dimensin. Tenemos que

C,

posteriormente demostraremos que para esa

dimensin la medida de Hausdor de

es 1, por lo que necesariamente esa

es la unin de dos copias a escala 1/3 de s mismo, luego

aplicando la medida de Hausdor a dichos subconjuntos, tenemos que:

1 s H (C) = H s 2
Suponiendo que

1 C 3

1 3

H s (C)

H s (C) > 0

tenemos que

1 = 2
luego

1 3

log 2 log 3 s Ahora veamos que para este valor de s, H (C) = 1. Dado > 0, consideramos la familia de intervalos {Ekj } de Ck que consk k tituyen un recubrimiento de C con j = 1, 2, ..., 2 y k tal que 3 < , se s=
verica que

2k s H

(C)
i=1

|Ek,i |s =

2 3s

=1

24

tomando el lmite cuando Falta ver que

0, H s (C) 1 H (C) 1. Para calcular la


s

medida de Hausdor es equi-

valente utilizar recubrimientos abiertos que cualquier tipo de recubrimiento (la demostracin puede verse en un recubrimiento abierto de son intervalos y como

C,

como trabajamos en

www.matgazine.tk ), luego tomamos {Ij } R podemos suponer que


{Ij : j = 1, 2, ..., m}

es compacto por ser interseccin de compactos nos

podemos quedar con un subrecubrimiento abierto nito

[5].

> 0 arbitrariamente pequeo podemos alargar los intervalos Ij , Ij si es necesario de tal manera que los extremos de estos estn fuera de C j {1, 2, ..., m}, pues el complementario de C en R es un conjunto k denso. Adems k N tal que 3 < . Cada intervalo {Ek,i } con i = k k 1, 2, ..., 2 , al tener longitud 3 , est contenido en al menos un intervalo Ij .
Dado obteniendo Veamos que

|Ek,i |s |Ij |s , j = 1, 2, , m
Ek,i Ij

Ek,i en Ij . Volviendo atrs en las iteraciones de la construccin de C , todos los Ek,i provienen de dos inter valos originarios contenidos en Ij , llammosles Ep y Eq , con p k y q k , pertenecientes a la iteracin p-sima y q-sima, respectivamente. Sea U el complementario en Ij de Ep Eq . Claramente, |U | mx{||Ep |, |Eq }, por lo a 1 que |U | (|Ep | + |Eq |). Por tanto, 2
Fijamos j y supongamos que hay ms de un

1 3 |Ij |s = (|Ep | + |Eq | + |U |)s ( (|Ep | + |Eq |))s = 3s ( (|Ep | + |Eq |))s = 2 2 1 1 = 2 ( (|Ep | + |Eq |))s 2 (|Ep |s + |Eq |s ) = |Ep |s + |Eq |s 2 2 s pues al ser x una funcin convexa, la imagen del punto medio es mayor o
igual que el punto medio de las imgenes. Por otro lado,

|Ep |s = (3 |Ep+1 |)s = 2|Ep+1 |s = |Ep+1 |s + |Ep+1 |s


y aplicando esto un nmero nito de veces, llegamos a que

|Ep |s =
Ek,i Ep
de donde se sigue que

|Ek,i |s , |Eq |s =
Ek,i Eq

|Ek,i |s

|Ij |s |Ep |s + |Eq |s =


Ek,i Ij

|Ek,i |s

25

como queramos ver. Finalmente,

2k

|Ij |
j=1 j=1 Ek,i Ij

|Ek,i |
i=1

|Ek,i |s = 2k 3k

2 3s

=1

Variantes del conjunto de Cantor


Ahora vamos a jugar un poco con el conjunto de Cantor y variantes suyas. Estas primeras variantes se basan en, elegido

k N, k 2,

iterar

innitas veces un mismo paso en los intervalos resultantes a partir del intervalo

[0, 1] R

de forma que obtengamos conjuntos autosemejantes.

La primera variante que vamos a trabajar consiste en, elegido de cada intervalo cerrado uno abierto de longitud zando en el intervalo obtenemos el conjunto

que Dk es autosemejante a escala 1 1 k 1 = 1 2k y tiene dos copias de s mismo, con la frmula que hemos 2 2 deducido anteriormente 1 log 2 s= 1 1 log 2 2k

[0, 1] Dk . Observamos

k , eliminar 1 de su longitud, comenk R. Iterando este proceso n veces e intersecando

Figura 2: Primeras iteraciones de

D5 .

Para la segunda variante en vez de quitar de cada intervalo uno de longitud 1 2 de su longitud, quitamos uno de longitud 1 k k con lo que en los extremos 1 tenemos intervalos de longitud k . Llamamos Ek a dicho conjunto que tiene 1 dos copias de s mismo a escala k , luego su dimensin de Hausdor es

s=
La siguiente variante,

log 2 log k k subin1 de forma alternada k

Fk ,

consiste en dividir cada intervalo en

tervalos iguales y eliminar subintervalos de longitud

26

Figura 3: Primeras iteraciones de

E4 .

k es impar, entonces permanecen 1 intervalos de longitud , luego su dimensin de Hausdor es k


dejando al que contiene al nmero 0. Si

k+1 2

s=
Si

log

2 k+1 1 log k

log k+1 2 log k

es par, dejamos a la derecha dos intervalos para no eliminar el nmero

k 1 2 1 copias a escala k y una copia del doble de tamao que las dems. luego aplicando la propiedad de la homotecia
1. En este caso el conjunto contiene de la medida de Hausdor

k 1 2
como

1 k

H s (Fk ) +

2 k

H s (Fk ) = H s (Fk )

H s (Fk ) = 0

k 1 = k s 2s 2 no podemos despejar s en funcin de k , pero sabemos dimensin de Hausdor de Fk satisface dicha expresin.

que el valor de la

Figura 4: Primeras iteraciones de

F4 .

Por ltimo, vamos a ver la construccin del conjunto de Smith-VolterraCantor o conjunto gordo de Cantor. Denotamos por mos del intervalo

dicho conjunto. Parti-

1 4 y obtenemos

V1 = [0, 1] R,

eliminamos el intervalo central de longitud

V2 .

En cada intervalo de

V2 ,

eliminamos el intervalo central

27

Figura 5: Primeras iteraciones de

F5 .

de longitud

24 .

En general, en cada uno de los

2n1

intervalos de

Vn

elimi-

namos el intervalo central de longitud intersecando, obtenemos

2n

, iterando

veces este proceso e

V.

Observamos que la medida de

es

2n

22n+2 n=0

=1

1 1 1 =1 = 2n+1 2 2 n=0

Este conjunto se emplea en la construccin de la funcin de Volterra, curiosa en el anlisis matemtico por tratarse de una funcin derivable, con derivada acotada, pero cuya derivada no es integrable en el sentido de Riemann, por lo que desafa el Teorema Fundamental del Clculo.

Figura 6: Conjunto de Smith-Volterra-Cantor.

Referencias
[1] M. Guzmn, [et al.], Estructuras fractales y sus aplicaciones. Barcelona, Labor D.L.1993 [2] K.J. Falconer, The Geometry of fractal sets. Cambridge University Press, 1985 [3] K. Falconer, Fractal geometry : mathematical foundations and applica-

tions.
John Wiley and Sons, 1999

28

[4] C. Shaver, An Exploration of the Cantor Set. Rockhurst University, 2009 [5] Mazn Ruiz, Jos M. Mazn Ruiz, Jos M. Madrid : McGraw-Hill, Interamericana de Espaa, 1997

Irene Gonzlez y Juan Rojo

La paradoja de Banach-Tarski

Introduccin
Imagina un puzzle que al deshacerlo y hacerlo en otro orden obtuvieras dos veces el mismo puzzle. La paradoja de Banach-Tarski arma que esto se puede hacer con la bola. Este resultado desafa nuestra intuicin geomtrica y por ello es uno de los teoremas ms controvertidos de la historia de la matemtica: para muchos es un claro ejemplo de que la matemtica pura no tiene conexin con la realidad y para otros es una razn ms que suciente para dejar de utilizar el axioma de eleccin. El estudio de la paradoja ha tenido adems consecuencias importantes en teora de la medida, teora axiomtica de conjuntos y teora de grupos. Sin embargo, el objetivo de este artculo no es tratar esos temas, sino demostrar el teorema. Para ello, empezaremos con unas deniciones bsicas.

Denicin. Unin disjunta: La unin de conjuntos disjuntos. Utilizaremos


el signo .

Denicin. Esfera de radio


una esfera unitaria la

centrada en el origen. Es decir

r: es la supercie esfrica en tres dimensiones {(x, y, z)| x2 + y 2 + z 2 = r2 , (x, y, z) R3 }. A denotaremos S . {(x, y, z)| x2 +
3

Denicin. Bola: es la esfera unitaria con su interior. Es decir

y + z 1, (x, y, z) R }.

La denotaremos

B.

Denicin. Movimiento rgido directo: Es una transformacin lineal que


preserva las distancias y la orientacin. Por lgebra lineal sabemos que es la composicin de una traslacin y una rotacin.

29

Denicin. Rotacin de la esfera: Es un movimiento rgido directo que lleva


la esfera a s misma. Es decir, una rotacin cuyo eje pasa por el origen.

Denicin. Grupo de transformaciones de la esfera: las rotaciones de la


esfera forman un grupo con la composicin. A este grupo lo llamaremos
Dada una transformacin

SO3 .

y un conjunto A de manera que puede acA denotamos por A = {a| a A}. Es decir, el conjunto que se obtiene al aplicar a todos los elementos de A. Si por ejemplo, el conjunto es B la bola unidad, y es trasladar una unidad en el eje de las X , tenemos que B es trasladar toda la bola una unidad en el eje de las X , ie la bola unidad centrada en el (1, 0, 0)
tuar sobre los elementos de Ya estamos en condiciones de enunciar la Paradoja de Banach-Tarski:

Teorema 2 (Paradoja de Banach-Tarski). Existen naturales

n,

conjuntos

Ai

de

disjuntos dos a dos para j n

1in
y

gi
n

movimientos rgidos directos

de manera que
i=1

Ai = B ,
i=1

gi Ai = B

gi Ai = B
i=j+1

Es decir, la bola se puede partir en una cantidad nita de subconjuntos disjuntos de manera que al mover los primeros obtenemos la bola de nuevo y al mover los segundos obtenemos la bola de vuelta. La paradoja se puede conseguir con 5 subconjuntos. Sin embargo, esa demostracin es demasiado intrincada, asi que aqu lo haremos con una cantidad mayor.

Demostracin. La losofa de la demostracin es que cuanto ms sencillo


es un objeto ms fcil es demostrar que cumple una cierta propiedad. Con esta idea en mente, dividiremos sucesivamente los objetos con los que tenemos que trabajar en objetos ms sencillos. All demostraremos resultados anlogos a los que queremos demostrar y despus encontraremos la forma de elevar esos resultados a los objetos ms complejos. La bola es la unin de esferas de radios

0<r 1

unin el origen. Una

vez hayamos demostrado un resultado anlogo en la esfera, ser sencillo extenderlo a toda la bola. La esfera a su vez, la podemos ver como una unin innita de hilos cada uno con una estructura muy sencilla: la de un grupo libre de orden dos. Empezaremos por explicar qu es un grupo libre de orden dos y las propiedades que nos interesan. En particular, veremos que este grupo es paradjico. A continuacin elevaremos esta propiedad a la esfera, comprobando que tambin es paradjicas.

30

Grupo libre de orden 2


En un Grupo libre de orden 2, al que llamaremos dos generadores:

para abreviar, hay

b.

Los elementos del Grupo libre son todas las palabras etc pertenecen todas a

nitas que se pueden hacer con los generadores y sus inversos. De esta manera,

ab, a, aa1 , abbabbbab1 aaa1 ,


es la palabra vaca.

y tambin

que

Dos palabras diferentes representarn al mismo elemento si y slo si podemos pasar de una a otra cancelando o aadiendo una letra con su consecutiva

aa1 b es el mismo elemento que b y que bbb . En particular ab es distinto de ba. Si no podemos cancelar ningu1 1 na letra ms, como en ababab b a, diremos que la palabra es reducida. Es
si una es la inversa de la otra. Por ejemplo,

inmediato comprobar que todo elemento tiene slo una palabra reducida.

W (g) formados por los eleg . Por ejemplo, W (a) son 1 las que empieza por a como ab, ab aaa1 , pero no aa1 b pues la forma reducida de este ltimo es b que no empieza por a.
Ms adelante, nos sern tiles los conjuntos mentos tales que su palabra reducida empieza por Hasta ahora slo hemos hablado de cmo son los elementos del grupo. Falta denir una operacin. sta va a ser concatenar.

a b = ab, abba a1 b = abbaa b = abbb. Necesitamos un elemento neutro: la palabra vaca ab = ab pues no estamos concatenando nada. Tambin un inverso: abbab1 ba1 b1 b1 a1 = . Es inmediato comprobar que toda palabra tiene inverso,
1
tan slo hace falta escribir los inversos de cada letra en el orden inverso. Por ltimo, tenemos que se cumple la propiedad asociativa. La demostracin de ello no es muy profunda, aunque s laboriosa. Por ello no la reproduciremos aqu, aunque el lector la puede encontrar en [1]. Finalmente, es importante para la demostracin observar que sobre s mismo. Para todo elemento

acta

de

tenemos la aplicacin

que acta

concatenando por la izquierda. Es decir:

g : F F : a ga a(b) = ab, abb(b1 a) = aba. Si tenemos un conjunto A y una palabra g de F , entonces podemos denir el conjunto gA = {ga|a A}. Por n 1 ejemplo, si A = {a |n > 0} entonces a A = {an |n 0}
Por ejemplo, Todo lo anterior ha sido introduccin al grupo libre. Es aqu donde de verdad empieza la demostracin de la paradoja. Si recordamos el enunciado del teorema tenamos dos estructuras bien diferenciadas. Por un lado, estaba el conjunto, la bola, que actuaba como puzzle. Todos los subconjuntos (las piezas) provenan de este conjunto. Por otro lado, tenamos el grupo de movimientos rgidos, que nos daba los movimientos que

31

podamos realizar. Podramos expresarlo diciendo que la bola es paradjica con respecto al grupo de movimientos rgidos, aunque preferimos dejar la denicin formal de paradjico para ms adelante. La primera pieza clave de la demostracin ser el hecho de que el grupo libre de orden 2, F, es paradjico con respecto a s mismo:

Lema 3. Existen naturales


para

j gi

y de

n F

y conjuntos

Ai

de n

disjuntos dos a dos j

1in

y elementos

de manera que
i=1

Ai = F ,
i=1

gi Ai = F

y
i=j+1

gi Ai = F

1 Demostracin. Estos conjuntos van a ser A1 = W (a), A2 = W (a ), A3 = 1 1 1 1 1 1 1 W (b){1, b , b b , . . .} y A4 = W (b )\{1, b , b b , . . .}. Estos cuatro

Ai = F . Si comprobamos que aA2 = bA4 = A1 A2 A4 , tendremos la segunda parte del teorema tambin, con g1 = , g2 = a, g3 = , g4 = b. Veamos los cuatro contenidos:
conjuntos son disjuntos dos a dos y

A2 A3 A4
1.

aA2 A2 A3 A4 : Si una palabra reducida g pertenece a uno de A2 , A3 o A4 , no empieza por a, por lo que en la palabra a1 g la a1 no se 1 cancela y a g es reducida. As, a1 g pertenece a A2 y g est contenida en aA2 . F \ A1 = A2 A3 A4 hay que comprobar aA2 pertenece A1 . Si g A2 es una palabra 1 1 reducida, entonces empieza por a por lo que g = a g donde g es una palabra reducida que no empieza por a pues sino se cancelara. Entonces, la forma reducida de ag es g que no empieza por a y por tanto no puede estar en A1 .
Como que ninguna palabra de

2.

aA2 A2 A3 A4 .

3.

bA4 A1 A2 A4 . Al igual que en el primer caso, los elementos de A1 , A2 y A4 no empiezan por b, por tanto, si su palabra reducida es g , b1 g tambin es reducida y b1 g A4 , as que g bA4 . bA4 A1 A2 A4 .
De nuevo como en el segundo caso, hay que demostrar que ningn elemento de

4.

A3

est en

bA4 .

Un elemento de

A3

es

tal que su palabra reducida empieza por

o que pertenece al conjunto

{1, b1 , b1 b1 , . . .}. Veamos que no se da ninguno de los dos casos. 1 Un g A4 es de la forma g = b g con g reducida tal que no empieza 1 1 1 por b y g = 1, b , b b , . . .. Por lo que bg = g y hemos visto que g justamente no puede estar en A3 , as que tenemos el contenido.
32

Puesto que se dan los cuatro contenidos, obtenemos que respecto a s mismo: dos a dos.

es paradjico con

F = A1

aA2

F = A3

bA4

siendo los

Ai

disjuntos

La esfera
Como ya habamos anunciado en la introduccin, nuestro siguiente paso ser partir la esfera en hilos de manera que cada uno se comporte como un grupo libre de orden 2 para elevar despus la propiedad de ser paradjico desde

hasta la esfera. En general, para conseguir la estructura de un grupo

en un conjunto hay que conseguir que ese grupo acte sobre el conjunto. Ese ser nuestro siguiente paso. Para poder enunciar el resultado clave en la demostracin, damos la siguiente denicin:

Denicin. El grupo generado por dos rotaciones


menor grupo que contiene a

de la esfera es el

y a

Es decir, sus elementos son todas las rotaciones que se pueden conseguir componiendo

, , 1

el crculo de la

1 : , 1 , etc. Para abreviar omitiremos composicin: = .


y

La forma que tienen los elementos de este grupo recuerdan a los elementos del grupo libre de orden 2. Sin embargo, aqu hay una diferencia enorme, a

priori un mismo elemento se podra escribir de dos maneras diferentes. Por


ejemplo, si consideramos como

o 180 la esfera tambin alrededor del eje Z. Tenemos que que consiste en o o girar 30 y despus 180 es lo mismo que que consiste en girarlos en el otro o orden. Tambin es girar 360 que es lo mismo que no haber girado, por
tanto

girar 30

o la esfera en torno al eje Z y

girar

es la identidad. Si recordamos la denicin de grupo libre de orden

2, dos palabras eran iguales si y slo si su palabra reducida era la misma. Como en este caso la palabra reducida de

no es

tenemos que ste no

es un grupo libre de orden 2. Sin embargo, tenemos el siguiente teorema:

Teorema 4. Existen dos rotaciones de la esfera


generan es un grupo libre de orden 2.

tales que el grupo que

Demostracin. Vamos a seguir la demostracin de [2]. De hecho,

rotaciones antihorarias en los ejes Z y X respectivamente, con ngulo

y son 1 arc cos 3 .

Para comprobar que el grupo que generan es un grupo libre de orden 2, hay que comprobar que una palabra reducida no trivial nunca es la identidad.

33

Tenemos

son transformaciones rgidas de

R3

as que vienen dadas

por matrices:

1 =

1 3 22 3

2 2 3 1 3

0
1

0 0 0

1 = 0 0

0 0 1

1 3 2 2 3

2 2 3 1 3

Una palabra en

es la identidad si y slo si

1 w

lo es. As, es suciente

comprobar que las palabras reducidas no triviales que acaban por la derecha

1 ,
Sea

no son la identidad. Vamos a demostrarlo por reduccin al absurdo. palabra reducida no trivial que termine en de la forma

que

w es una w(1, 0, 0) es

(a, b 2, c)/3k

1 .

Armamos

donde

a,b, c

son enteros y

no

es divisible por 3. Si conseguimos probar esto habremos terminado, pues si

w
es

fuera la identidad deberamos tener

w(1, 0, 0) = (1, 0, 0),

que no es de esta

forma. La demostracin ser por induccin en la longitud de

entonces

tiene que ser

y por tanto

w. Si la longitud w(1, 0, 0) = (1, 2 2, 0)/3 as

que tenemos el caso base. La induccin la haremos en dos partes. Primero

a,b, c son enteros y a continuacin que b no es divisible por 3. w w = 1 w o w = 1 w , donde por la hiptesis de induccin w (1, 0, 0) = (a , b 2, c )/3. Multiplicando w (1, 0, 0) por las matrices de 1 o 1 tenemos que w(1, 0, 0) es de la forma requerida con a = a 4b , b = b 2a , 2c , c = c 4b en el segundo. c = 3c en el primer caso y a = 3a , b = b Para comprobar que b nunca es divisible por 3 distinguimos cuatro casos, 1 1 segn w sea v , 1 1 v , 1 1 v y 1 1 v , donde v puede ser la 1 palabra vaca (siempre y cuando w siga terminando en ). Si a , b y c son los enteros que salen al calcular v(1, 0, 0) (en el caso de que v sea la 0 palabra vaca tenemos que v(1, 0, 0) = (1, 0 2, 0)/3 por lo que tambin tiene
veremos que es de la forma sentido), y utilizando la misma notacin que en el prrafo anterior tenemos para el primer caso que el segundo caso,

b = b 2a y adems 3 divide a a pues a = 3a . Y en 2c y 3 divide a c . Por induccin, 3 no divide a b as que tampoco divide a b. Para los otros dos casos, tenemos que b = 2b 9b . Por ejemplo, en el cuarto caso, b = b 2c = b 2c 8b = 2b 9b . En el tercer caso se procede de igual manera. Por induccin b no era divisible entre 3 as que b tampoco, w no puede ser la identidad y hemos acabado. b=b
El teorema que acabamos de probar nos dice que el grupo de orden 2 acta

sobre la esfera, siendo los generadores de

los

mo conseguimos ahora que la esfera tenga la estructura de

y de la demostracin. CF ? Centrndonos

34

en cada rbita. Para todo punto

x de la esfera, tenemos asociada la rbita F x = {f x| f F }, que es uno de los hilos que venimos anunciando. Cuando x no sea un punto jo de F no tendremos problema para conseguir la paradoja,
trabajando al mismo tiempo en todas las rbitas que no tienen puntos jos, casi tendremos que la esfera es "paradjica". Esto es lo que dice el siguiente lema:

Lema 5. Existe un conjunto contable


naturales

de la esfera

S , de manera que S\D

es

paradjico con respecto al grupo de rotaciones de la esfera. Es decir, existen

j y n y conjuntos Ai de S \ D disjuntos dos a dos para 1 i j n rotaciones de la esfera de manera que i=1 Ai = S \ D , i=1 gi Ai = n y i=j+1 gi Ai = S \ D gi
Demostracin.

ny S\D

la identidad) de

D va a ser el conjunto de puntos jos no triviales (obviando F , es decir, los x de la esfera de manera que existe un g F distinto de la identidad de manera que gx = x. Como F estaba generado por dos rotaciones, F es contable. Como todos los elementos de F son rotaciones
de la esfera y cada rotacin que no es la identidad deja exactamente dos puntos jos, tenemos que

es contable.

tambin est:

x S \ D, entonces toda la rbita F x S \ D. Vemoslo por reduccin al absurdo, si existe g F de manera que gx S \ D quiere decir que gx es un punto jo no trivial de F : existe f F \ {id} de manera que f gx = gx, pero entonces g 1 f gx = x y por tanto x tambin es punto jo no trivial de F y x S \ D , contradiccin. 1 Tambin f x = gx si f = g , pues tendramos g f x = x, por lo que g = f . Si recordamos la demostracin de que F era "paradjico", tenamos que los 1 subconjuntos con los que se consegua eran A1 = W (), A2 = W ( ) , A3 = 1 1 1 W () {1, , , . . .}, A4 = W (1 ) \ {1, 1 , 1 1 , . . .}. Si jamos un x S \ D, como x no es punto jo no trivial tenemos que A1 x, A2 x, A3 x y A4 x son conjuntos disjuntos
Lo siguiente que observamos es que si

de la esfera. Por tanto tenemos que la unin es disjunta:

Ai x = F x.
i=1

Por

el lema 3 tambin tenemos que con

g1 A1 x

g2 A2 x = F x

g3 A3 x

g1 = , g2 = , g3 = :: , g4 = , g4 A4 x = F x. Es decir, podemos reproducir

la paradoja en una de las rbitas. Lo que hay que hacer ahora es lo mismo en todas las rbitas a la vez. Para ello utilizamos el axioma de eleccin que dice lo siguiente:

Axioma de Eleccin. Dada una familia


existe un conjunto

de manera que

C de conjuntos |C C | = 1 .
35

donde

Intuitivamente, lo que dice el axioma de eleccin es que si vemos los conjuntos como sacos podemos sacar un elemento de cada saco al mismo tiempo. Vamos a utilizar el axioma de eleccin sobre las rbitas. Por el axioma de eleccin existe un conjunto un punto. Los conjuntos

f (C) g(C) =

es

C que corta a cada rbita en exactamente {g(C)| g F } son una particin de S \ D es decir equivalente a que f = g y g(C) = S \ D. La paradoja
gF

se obtiene con los conjuntos

Ci = Ai C . Pues como es una particin, Ci = Ai C = gF g(C) = S \ D, y tomando g1 = , g2 = , g3 = , g4 = , tenemos que g1 C1 g2 C2 = S \ D y g2 C2 g3 C3 = S \ D, por tanto, S \ D es paradjica con respecto a F . Como los elementos de F eran elementos del grupo de rotaciones de la esfera, S \ D es paradjica con respecto al grupo de
rotaciones de la esfera, que es lo que queramos demostrar.

Para poder demostrar el teorema que tenamos como objetivo en este apartado, ie que la esfera es "paradjica", tenemos que encontrar la manera de "deshacernos"del conjunto con el siguiente lema:

de puntos jos de

F.

Eso es lo que conseguimos

Lema 6. Si
conjuntos y

fA

S es la esfera y D S un subconjunto contable, existen dos A y B y dos rotaciones de la esfera f y g de manera que A B = S gB = S \ D.

Es decir, partimos la esfera en dos trozos y al moverlos conseguimos que desaparezca el conjunto contable.

Demostracin. Para la demostracin necesitamos una rotacin 2


de manera que

de la esfera

D, (D), (D) . . .

sean disjuntos dos a dos. Esta

existe

por cardinalidad. En efecto, tomamos un eje corte a

que pase por el origen y que no

D y llamamos Kn al conjunto de ngulos tales que n (D) D = donde es una rotacin de ngulo . Como D es contable y el eje l no corta a D , Kn es contable. Por tanto, [0, 2] \ Kn es no vaca. Tomando en ese n conjunto, tenemos que (D) D = para todo n, lo que es equivalente a n m que (D) y (D) sean disjuntos para n y m distintos. Los conjuntos que tomamos son A = D (D) 2 (D) . . . y B = S \ A. Evidentemente A B = S . Adems, (A) = A\D . Por tanto (A) B = S \D
como queramos.

Ya estamos en disposicin de demostrar el que era nuestro objetivo en esta seccin.

36

Teorema 7 (Paradoja de Hausdor ). La esfera es paradjica con respecto


al grupo de rotaciones de la esfera. Es decir, existen naturales conjuntos disjuntos dos a dos para j n esfera de manera que i=1 Bi = S , i=1 gi Bi

Bi

de

j y n y sub1 i n y gi rotaciones de la =S y n i=j+1 gi Bi = S

El teorema es consecuencia directa de los dos lemas anteriores y la demostracin es sencilla. Sin embargo, vamos a aplazarla hasta despus de un par de deniciones y una proposicin asociada. Las nuevas deniciones permitirn expresar los resultados probados de una manera ms compacta y slo eso ya justicara introducirlas en este artculo. Sin embargo, se ha preferido no dar la denicin hasta ahora para que el lector se pudiera familiarizar con los conceptos, pero no aplazarlo ms porque con ello conseguiremos aligerar signicativamente la notacin en la demostracin. Despus de todo el trabajo que hemos hecho, las siguientes deniciones no deberan causar problemas.

Denicin. Supongamos que

G es un grupo que acta en X . Diremos que G-paradjico si existen naturales j y n, conjuntos Xi de X disjuntos n dos a dos para 1 i n y gi elementos de G de manera que i=1 Xi = X , j n i=1 gi Xi = X y i=j+1 gi Xi = X
es
A la vista de la nueva denicin, comprobamos que lo dicho informalmente era ciertamente lo que se estaba haciendo: El objetivo del artculo es demostrar que la esfera es paradjica con respecto al grupo de transformaciones rgidas. En cuanto a los resultados que ya hemos demostrado, el primer lema armaba que

era

conjunto

contable de la esfera

de manera que

respecto al grupo de rotaciones de la esfera

F -paradjico y que exista un S \ D era paradjico con (SO3 ).

La segunda denicin que necesitaremos es la siguiente:

Denicin. Supongamos que


Diremos que

G es un grupo que acta en X y A, B X . A y B son G-equidescomponibles si existe un natural n, Ai A, Bi B y gi G para i = 1, . . . , n de manera que A = n Ai , B = n Bi i=1 i=1 y Bi = gi (Ai ). La notacin que utilizaremos es A G B .

Por ejemplo, si consideramos X la recta real, G el grupo de traslaciones, A = {1, 2, 3, 4} y B = {1, 2, 5, 6} tenemos que A G B , una posible manera con n = 2, A1 = {1, 2}, A2 = {3, 4}, B1 = {1, 2}, B2 = {5, 6}, g1 = id y g2 (x) = x + 2. Puesto que el ltimo lema arma que existen A y B de manera que A B = S y A B = S \ D . Se puede reformular diciendo que S y S \ D son SO3 -equidescomponibles.

37

Con esta denicin, se puede expresar que que existen conjuntos

sea

de

de manera que

G-paradjico diciendo B = X , A G X y

B G X .
Se ha utilizado la notacin respecto a

para denotar la equidescomponibilidad con

pues se trata de una relacin de equivalencia. Comprobmoslo:

1. Reexiva. Basta tomar que

A=

1 i=1 Ai

A1 = 1 i=1 Bi y

B1 = A y como g B1 = idA1 .

la identidad

id. Tenemos

2. Simtrica. Basta tomar los inversos, ie si un natural

A G B quiere decir que existe n, Ai A, Bi B y gi G para i = 1, . . . , n de manera n n que A = i=1 Ai , B = i=1 Bi y Bi = gi (Ai ). Para ver que B G A 1 1 basta tomar como gi = gi y tenemos que gi (Bi ) = gi (Bi ) = Ai como
queramos.

3. Transitiva. Hay que tomar la interseccin de los conjuntos. Si tenemos

Ai y Bi para 1 i n gi y que B G C se consigue con unos conjuntos Bj y Cj para 1 j m y transformaciones hj . Para obtener A G C 1 1 basta tomar como conjuntos Aij = gi (Bi Bj ) = Ai gi (Bj ) y Cij = hj (Bi Bj ) = hj (Bi ) Cj y transformaciones fij = hj gi para 1 1 i n y 1 j m. En efecto fij (Aij ) = hj gi (gi (Bi Bj ) = n m 1 hj (Bi Bj ) = Cij y i=1 j=1 Aij = i=1 Ai = A. Por ejemplo, tomando A = {1, 2, 3, 4} y B = {1, 2, 5, 6} como antes y C = {1, 5, 9, 13}. Si B1 = {1, 5}, B2 = {2, 6}, C1 = {1, 5}, C2 = {9, 13}, 1 h1 = id y h2 (x) = x + 7, tenemos que A22 = A2 g2 (B2 ) = {3, 4} {0, 4} = {4}, C22 = h2 ({5, 6}) {9, 13} = {12, 13} {9, 13} = {13} y f22 (x) = h2 g2 (x) = h2 (x + 2) = x + 9, anlogamente con el resto de los Aij . De la demostracin de la transitividad se obtiene que si A G B con n conjuntos y B G C con m conjuntos se puede conseguir que A G C a lo sumo con mn conjuntos (eventualmente menos)
que se consigue con unos conjuntos y transformaciones La proposicin que queremos demostrar es la siguiente:

A G B

Proposicin 8. Supongamos que


juntos equidescomponibles de tambin lo es

X,

ie

G acta sobre X , y sean E y E subconE G E . Si E es G-paradjico entonces

Antes de hacer la demostracin, observamos que esta proposicin tiene como consecuencia directa que la esfera de rotaciones de la esfera un conjunto contable

es paradjica con respecto al grupo

SO3 .

En efecto, ya tenemos demostrado que existe

tambin tenemos que para todo conjunto contable

D de la esfera de manera que S \D es SO3 -paradjica. Y D de la esfera S SO3 S\D.


38

Demostracin. Tan slo hace falta recordar que el hecho de que


de manera que

fuera

paradjico se poda expresar diciendo que existan conjuntos disjuntos

GB

B = E , A G E y B G E . Vamos a comprobar, que E G E , existen conjuntos A y B disjuntos de E de manera que A B = E , A G A y B G . Con esto habramos acabado, pues tendramos que A G A G E G E y B G B G E G E y por la transitividad de la equidescomponibilidad, A G E y B G E , es decir, que E es G-paradjico. Comprobmoslo: por la denicin existan unos conjuntos Ai de E disjuntos dos a dos cuya unin era E y Bi de E y transformaciones gi de manera que gi Ai = Bi . Los conjuntos A y B que buscamos son A := gi (Ai A) y B := gi (Ai B). Se comprueba rpidamente lo que se quera: A B = , A B = (gi (Ai A) gi (Ai B)) = gi (Ai E) = Bi = E , A G A y B G B . A
como tenemos que

Teorema 9 (Paradoja de Hausdor ). La esfera

es

SO3 -paradjica,

donde

SO3

es el grupo de rotaciones de la esfera.

Demostracin. Como ya hemos dicho anteriormente es consecuencia de lo


proposicin 8 . Tenamos que

D,
es

y que

S\D

era

S SO3 S \ D para un cierto conjunto contable SO3 -paradjico. Por la proposicin anterior, tambin lo 2 4 2 = 16

S.
El lector interesado podr comprobar que se han utilizado

piezas para la paradoja. El dos viene de quitar y el

D,

el

de aplicar la paradoja

de volver a colocar

D.

La Bola
En esta seccin demostramos nalmente el que era nuestro objetivo, la paradoja de Banach-Tarski. Empezamos por recordar qu queramos probar:

Teorema 10 (Paradoja de Banach-Tarski). La bola


donde

es

SE(3)-paradjica,

SE(3)

es el grupo de movimientos rgidos directos del espacio.

Si la bola unidad no tuviera el origen, la paradoja de Banach-Tarski sera automtica:

Lema 11.

B\0

es

SE(3)-paradjico,

donde

es el origen.

Demostracin. Por la paradoja de Haussdor que ya hemos demostrado, tenemos que la esfera unidad juntos

SO3 -paradjica es decir, existen unos conjuntos disj n

Ai para 1 i n de manera que

Ai = S ,
i=1

gi Ai = S y
i=j+1

gi Ai =

39

S,

S r a la esfera de radio r 2 r centrada en el origen, es decir {(x, y, z)| x + y + z = r }, tenemos que S r tambin es SO3 -paradjica. Basta tomar como Ai = {(rx, ry, rz)| (x, y, z) Ai } y como gi los mismos de la esfera, tenemos automticamente que los Ar i
donde

gi

son rotaciones de la esfera. Si llamamos

j
son disjuntos,

Ar = S r i

gi Ar = S r i
i=1

i = j + 1n gi Ar = S r . i

B \ 0 = S r para 0 < r 1, para ver que es SO3 paradjico basta tomar como Bi = Ar para 0 < r 1. Tenemos que los Bi i son disjuntos y que con los gi de antes hacen que B \ 0 sea SO(3)-paradjico. Como SO(3) es un subgrupo de SE(3) ya tenemos lo que queramos.
Finalmente, como Procedemos ahora de manera anloga al caso de la esfera.

Lema 12.

B\0

son

SE(3)

equidescomponibles,

B \ 0 SE(3) B .

Demostracin. Tomamos
Elegimos una rotacin

y un eje que no pase por el (0, 0, 0). de manera que n no sea la identidad para ningn n. Tomando A = 0 0 2 0 . . . y C = B \ A, tenemos que (A) = A \ 0, y por tanto B = A C y B \ 0 = A idC . Como es una isometra directa, tenemos que B \ 0 SE(3) B . Finalmente:

1 P = (0, 0, 2 )

Teorema 13 (Paradoja de Banach-Tarski). La bola


Demostracin.

es

SE(3)-paradjica.
por la

proposicin 8 tenemos que

B SE(3) B \ 0, como B \ 0 es SE(3)-paradjico, B es SE(3)-paradjico.

El lector atento podr comprobar que hemos demostrado que con

1622 =

48

piezas es posible conseguir la paradoja.

Unas ltimas palabras


Una vez demostrado el teorema, debemos reexionar cules son las consecuencias. El marco adecuado es el de la teora de la medida. Para poder desarrollar un anlisis ecaz es necesaria una buena denicin de integral y la experiencia muestra que un buen camino pasa por la denicin de una medida, ie un volumen de los conjuntos. Entre las propiedades que se le piden a este volumen, con la intencin de tener una buena teora de la integracin, es que al unir una cantidad contable

40

de conjuntos disjuntos, el volumen total sea la suma de los volmenes. Es fcil comprobar (Paradoja de Vitali) con el axioma de eleccin, que no es posible denir este volumen tan siquiera en

R.

Casi inmediatamente se plantea el problema de si existe una nocin de volumen a la que se le exija, entre otras condiciones, algo ms dbil: que la unin nita de conjuntos disjuntos tenga como volumen la suma de los volmenes. Curiosamente, este volumen existe en y en

R2 ,

pero no en

R3 ,

pues de haberlo no se podra dar la paradoja de Banach-Tarski, que acabamos de probar. La razn de ello es en gran parte debido a que el grupo de isometras de

R3

contienen un subgrupo libre de orden 2, mientras que los de

R y R2

no.

En este momento uno tiene dos opciones. La primera es contentarse con una nocin de volumen que sirve para prcticamente todos los conjuntos "tiles" mientras que la segunda es cuestionarse los fundamentos. Tanto en la demostracin de la paradoja de Banach-Tarski como de la paradoja de Vitali se ha utilizado el axioma de eleccin (AC) . As pues, se podra pensar que el axioma de eleccin juega un papel clave en estas demostraciones. Y en efecto as es: en 1964 Solovay demostr que era consistente con ZermeloFraenkel (ZF) que todos los conjuntos tuvieran volumen. Incluso si se peda que el volumen de la unin contable de conjuntos disjuntos fuera la suma de los volmenes. Aunque hubo ciertas esperanzas puestas en esta teora, no se vieron recompensadas, pues el anlisis es mucho ms potente si se utiliza el axioma de eleccin y es por ello que el marco axiomtico estndar es ZF+AC. La paradoja de Banach-Tarski plantea automticamente otros problemas, como cul es la cantidad mnima de piezas, que resulta ser 5. La prueba de ello utiliza el hecho de que si dos rotaciones de la esfera dejan jo los mismos puntos entonces conmutan. La demostracin del recproco, ie que no es posible con menos de 4, es relativamente sencilla y depende de argumentos geomtricos. Tambin nos podemos preguntar cules son los conjuntos que son equidescomponibles con respecto a

SE(3). Sorprendentemente resulta que

todos los conjuntos acotados con interior no vaco son equidescomponibles. En la literatura se suele ilustrar diciendo que un guisante se puede partir en una cantidad nita de trozos, reordenarlos y conseguir el sol. Para demostrar este resultado se utiliza una versin especial del teorema de Schroeder-Bernstein de teora de Conjuntos. Todos estos temas y muchos ms, como la relacionada con teora de grupos, se encuentra con mucho ms detalle en la obra enciclopdica sobre este tema

The Banach-Tarski Paradox, de Stan Wagon, que recoge todos los avances
hasta 1984 y que se encuentra en la biblioteca de la Facultad.

2 El lector recordar que la axiomtica ms usual es la de Zermelo-Fraenkel (ZF) con el axioma de eleccin: ZF+AC.

41

Referencias
[1] A. Hatcher, Algebraic Topology. Cambridge University Press, 2002 [2] S. Wagon, The Banach-Tarski Paradox. Cambridge University Press, 1985

Gabriel Furstenheim

42

Rayos y centellas!

A estas cosas osis llamar artculos? Joder, es que no os he enseado nada? A ver si con el mo se os pega algo...
43

reas y longitudes
Estas lneas estn escritas para divulgar entre los alumnos de la facultad un ejemplo obtenido en el ao 1890 por el matemtico polaco K.H.A. Schwarz, en el que encuentra, jado un cilindro de revolucin acotado inscritas en

M,

sucesiones

de supercies polidricas constituidas por tringulos cuyas reas tienden a

0,

y cuyas reas no convergen al rea de

M.

Esto no sucede si

sustituimos supercie por arco de curva diferenciable y supercie polidrica por curva poligonal, por lo que comenzamos exponiendo lo que sucede en el caso unidimensional.

Clculo de longitudes de arcos de curva.


Sea

: I = [a, b] Rp , t (1 (t), . . . , p (t)) C = (I) Rp cuya restriccin al intervalo abierto (a, b) es de clase C , es decir, cada i |(a,b) : (a, b) R es una
un homeomorsmo sobre el arco de curva

funcin derivable cuya derivada es una funcin continua. Una particin de I es un subconjunto I , donde cada tk < tk+1 , y la poligonal {(tk ) : 0 k n} mide de nito

P = {a = t0 , . . . , tn = b}

que tiene por vrtices los puntos

(, P) =
k=1
El nmero cin

(tk ) (tk1 ) .
se llama dimetro de la parti-

= m k+1 tk : 0 k n 1} n{t 0
cuando

P.

No parece descabellado pensar que si se eligen particiones

{Pm }mN

cuyos dimetros converjan a

m ,

la sucesin de longitudes

{ (, Pm )}mN C.

converja a un nmero real, al que se debe llamar longitud de

Y as sucede, efectivamente! Escribiendo, como es habitual en Matemticas, en el orden contrario al que uno piensa, lo que hace ms fcil seguir los pasos del argumento pero esconde cmo se ha llegado al resultado que uno se propone probar, se dene la longitud de

como la integral

(C) =
a
y vamos a demostrar que: Para cada dimetro menor que

(t) dt,

> 0 existe > 0 tal que para cada , se tiene | (C) (, P)| < .
44

particin

de

con

Para ello consideramos la funcin auxiliar

f : I p R, (t1 , . . . , tp )
que es continua y cumple

1 (t1 )2 + + p (tp )2 ,

f (t, . . . , t) = (t) . El cubo I p es cerrado y acotado en R , luego compacto. En consecuencia, f es uniformemente continua, por lo p que existe > 0 tal que para cada = (t1 , . . . , tp ) I con |ti ti | < para cada 1 i p se cumple que |f (t1 , . . . , tp ) f (t1 , . . . , tp )| < /(b a). Denotamos Ik = [tk1 , tk ] y aplicamos el Teorema del valor medio a cada restriccin i |Ik . Existe por tanto, para cada 1 i p, un punto ik (tk1 , tk ) tal que i (tk ) i (tk1 ) = i (ik )(tk tk1 ). Si E = {e1 , . . . , ep } es la base estndar p de R lo anterior se reescribe como
p p p

(tk ) (tk1 ) =
i=1
y esto implica que

(i (tk ) i (tk1 ))ei = (tk tk1 )


i=1

i (ik )ei ,

(, P) =
k=1

(tk ) (tk1 ) =
k=1

(tk tk1 )f (1k , . . . , pk ). k


. Se

Por el Teorema del valor medio del Clculo Integral existen puntos

(tk1 , tk )

con

1 k n,

tales que

tk tk1

(t) dt = (tk tk1 ) (k )

tiene entonces,

tk

(C) =
a

(t) dt =
k=1 n tk1

(t) dt
n

(tk tk1 ) (k ) =
k=1

(tk tk1 )f (k , . . . , k ).
k=1

Restando las expresiones de

(, P)

(C)

que acabamos de obtener resulta

| (, P) (C)| =
n

(tk tk1 )(f (1k , . . . , pk ) f (k , . . . , k ))


k=1

<

(tk tk1 )|f (1k , . . . , pk ) f (k , . . . , k )|


k=1 n

(tk tk1 )/(b a) = ,


k=1

45

como queramos demostrar. Por analoga con la denicin de longitud de un arco de curva, parece indicado denir el rea de una supercie acotada como lmite de las aproximaciones a la misma mediante supercies polidricas inscritas, de modo que el tamao de las caras de estas supercies tienda a cero. Sin embargo tal idea no es buena, como puso de maniesto el matemtico polaco K.H.A. Schwarz en 1890, quien construy aproximaciones polidricas a un cilindro de revolucin acotado cuyas reas no tienen lmite. Exponemos a continuacin este ejemplo.

El ejemplo de Schwarz

radio

M cuya base es una circunferencia de R y cuya altura es h. Su rea coincide con la del rectngulo de base 2R y altura h que se obtiene al cortarlo a lo largo de una de sus generatrices y desarrollarlo sobre un plano, esto es, Area(M ) = 2Rh. Vamos a aproximar M mediante supercies polidricas, cuyas caras son
Consideremos un cilindro vertical tringulos, inscritas en la supercie del modo siguiente. Fijamos dos enteros positivos utivos,

n.

Dividimos

en

rebanadas cortndola mediante

n1

planos horizontales, de manera que dos de dichos planos distan, si son consecque

C0 , . . . , Cn a las circunferencias obtenidas, de modo Ck y Ck+1 sean consecutivas. Diremos que Ck y Ck+1 son circunferencias adyacentes. Inscribimos a continuacin, en cada Ck , un polgono regular de m vrtices. Situamos estos vrtices en C0 de modo arbitrario; hecho esto los colocamos en C1 tras haberlos girado, con centro el centro de C0 , un ngulo /m, y luego levantarlos hasta C1 . Repetimos el proceso, construyendo un polgono regular de m vrtices en Ck+1 obtenidos despus de girar, con centro el de Ck , un ngulo /m los vrtices de Ck y levantndolos a continuacin hasta Ck+1 . Una vez acabado el proceso se une cada vrtice de Ck con los dos ms cercanos de cada circunferencia adyacente a Ck , de manera que se obtiene una supercie polidrica constituida por 2mn tringulos, todos ellos con el mismo rea. Denotamos Sm,n ese valor comn y vamos a calcularlo. Con las notaciones de la gura, d(A, D) = R sen(/m), mientras que, por el Teorema 2 2 2 de Pitgoras, d(C, D) = d(C, E) + d(E, D) . Ahora bien, d(C, E) = h/n, mientras que d(E, D) = R d(O, D) = R R cos(/m). En consecuencia,
Denotamos

h/n.

C0

sea la de la base y

Sm,n = d(A, D)d(C, D) = R sen(/m)


Al sumar el rea de los

h2 + R2 (1 cos(/m))2 . n2

2mn

tringulos que la constituyen obtenemos el

46

Figura 7:

Cilindro

rea

Pm,n

de la supercie polidrica inscrita, cuyo valor es

Pm,n = 2mnR sen(/m)

h2 + R2 (1 cos(/m))2 n2 = 2mR sen(/m) h2 + R2 n2 (1 cos(/m))2

= 2mRh sen(/m) 1 + R2 n2 (1 cos(/m))2 /h2 .


La Frmula de Taylor aplicada a la funcin

cos

en

nos dice que existe

f de clase C 2 cos = 1 /2 + 4 f ().


una funcin

en un intervalo abierto centrado en

tal que

Comprobaremos a continuacin que el valor del lmite de los nmeros

Pm,n

cuando

tienden a innito, e incluso la existencia de dicho lmite,

dependen de la relacin entre

n.

Supongamos, por ejemplo, que

n =

mk , donde k N+ . Entonces, l m m sen(/m) = , m

denotando

Qm,k = Pm,mk

resulta, puesto que

l {Qm,k }mN+ = 2Rh l m m

1 + R2 m2k 4 /4m4 h2

mN+

La eleccin de distintos valores de Por ejemplo, para

k proporciona muy distintos resultados. k = 1 se obtiene el resultado esperado, a saber, l m {Qm,1 }mN+ = 2Rh, pero para k = 2 se tiene m
m

l {Qm,2 }mN+ = 2Rh 1 + R2 4 /4h2 > 2Rh, m {Qm,k }mN+


47 es

y para

k>2

el lmite de la sucesin

+.

Esto obliga a denir el rea de una regn de la supercie contenida en la imagen de una parametrizacin por decreto como la integral del determinante de la primera forma fundamental de la supercie respecto de la parametrizacin escogida y, claro, a comprobar que esta denicin no depende de la parametrizacin. Dicha comprobacin se basa, exclusivamente, en el llamado Teorema del cambio de variable del Clculo Integral, y este hecho es uno ms de los que hace importante dicho teorema.

Jos Manuel Gamboa, Dpto de lgebra, Fac. CC. Matemticas. UCM.

48

Problemas y soluciones
Las soluciones a los problemas deben enviarse, preferiblemente, al encar(problemas.matgazine@gmail.com), en formato T X. Una forma alternatiE va es entregarlas en mano en la asociacin Lewis Carroll (ver direccin en la primera pgina). El plazo de entrega termina el 20 de Septiembre del 2011. Tambin solicitamos de los lectores problemas originales (con solucin) o problemas poco conocidos adecuadamente documentados. Para su publicacin se valorar su inters matemtico. Para incitar ms vuestra colaboracin, las personas que mandis la solucin que escojamos a uno de los problemas recibir el siguiente ejemplar de la revista gratuito. Asmismo, si proponis algn problema que sea seleccionado, tambin recibiris gratis el siguiente nmero. Los problemas estn organizados en orden creciente de dicultad, donde indicar que un problema es difcil y problemas abiertos o sugeridos sin solucin se indicarn con un asterisco gado de la seccin, Gabriel Frstenheim Milerud

que es especialmente complicado. Los


.

Problemas Problema 7.
Propuesto por Miguel Ambrona, estudiante de la UCM

Mientras pasan el rato tres amigos, todos acionados a los problemas de ingenio y perfectos lgicos, uno de ellos dice lo siguiente: -Estoy pensando en dos nmeros, ambos mayores o iguales que 2 y menores o iguales que 50. Samuel, a ti te voy a decir la suma de ambos y sin embargo a ti, Pedro, te voy a decir el producto. Y as lo hace, de forma que cada uno de ellos slo conoce el dato correspondiente. A continuacin se produce la siguiente conversacin: -Pedro, eres capaz de decirme qu dos nmeros estoy pensando? -Claro que no- dice Pedro. -Ya saba que no ibas a poder- arma Samuel. -Ah, pues entonces ya s cules son!- exclama Pedro con una sonrisa. -Pues entonces yo tambin!- concluye Samuel. Cules son los dos nmeros en los que estaba pensando el primero que habl?

Problema 8.
UAM

Propuesto por Diego Soler Polo, alumno de CC Fsicas en la

49

1. Existe algn espacio vectorial con dos normas denidas en l y una sucesin que converja con la primera norma pero no con la segunda? 2. Existe algn espacio vectorial con dos normas denidas en l y una sucesin que converja con ambas normas, pero siendo los dos lmites diferentes?

Problema 9.
Sea

Propuesto por la redaccin (Putnam)

una curva de Jordan, ie cerrada y que no pasa dos veces por el mismo

punto, en el plano. Demostrar que para todo punto curva, existen puntos del segmento que

en el interior de la

P1 y P2 en une P1 y P2 .

la curva de manera que

es el punto medio

Problema 10.
Sea

Propuesto por la redaccin n un entero par. Cuntos subconjuntos del conjunto {1, 2, . . . , n} se pueden

elegir si todos tienen una cantidad impar de elementos y la interseccin de dos cualesquiera tiene cardinal par?

Problema 11.
Jamn y Queso)

Propuesto por la redaccin (Teorema del bocadillo de

Sea un bocadillo de jamn y queso en el espacio. Demostrar que se puede cortar dejando la mitad de pan, jamn y queso a un lado y la otra mitad al otro. Por bocadillo de jamn y queso se entiende tres conjuntos

P, J

Q de volumen

nito en el espacio. Y por corte dividir el espacio en dos con un plano.

Soluciones Problema 1.
En un tablero de ajedrez de 88 quitamos dos esquinas opuestas. Es posible cubrir completamente el tablero con chas de domin de 21 sin que stas se solapen o se salgan del tablero?

Solucin enviada por Pablo Aguado, estudiante de la UAM.


No es posible. Cada cha de domin que se coloca sobre el tablero cubre dos casillas contiguas, una blanca y una negra, y como al quitar dos esquinas opuestas se quitan dos casillas del mismo color siempre quedarn en el tablero por cubrir dos casillas ms de un color que de otro.

Tambin resuelto por Disoluto Martnez

50

Problema 2.

En el stano del castillo, 7 gnomos guardan su tesoro. El

tesoro est detrs de 12 puertas, cada una de ellas con 12 cerraduras. Todas las cerraduras son distintas. Cada gnomo tiene llaves para algunas de las cerraduras. Tres gnomos cualesquiera tienen conjuntamente llaves para todas las cerraduras. Probar que entre todos los gnomos tienen por lo menos 336 llaves.

Solucin enviada por Pedro ngel Castillejo


Hay un total de 144 cerraduras. Vamos a probar que el menor nmero de llaves para que cada eleccin de 3 gnomos sume 144 llaves es 336. Esto no prueba que sea el nmero mnimo de llaves necesarias; lo que pruebo es que seguro que no van a ser menos de 336. Si cada gnomo tiene 48 llaves, entonces tienen

748 = 336 llaves, y cada 3 gnomos

3 48 = 144

llaves.

Supongamos que el primer gnomo tiene una llave menos, 47. Entonces, el resto de gnomos (salvo, por ejemplo, el 2 ) tendrn que tener una llave ms, 49, para que al elegir cualquier conjunto de 3 gnomos sumen al menos 144 llaves. Si hubiese dos gnomos distintos del primero con 48 llaves, al juntarse con el primero no llegaran a cubrir las 144 cerraduras. Por cada llave que deje de tener el primero, 5 de los restantes gnomos tendrn que tener una llave ms, por lo que la conguracin con menos llaves es la arriba considerada.

Nota de la redaccin: El mnimo de llaves que hacen falta para que tres gnomos
cualesquiera puedan abrir es en realidad mucho mayor. Si una llave no la tuvieran tres gnomos eligiendo a esos tres gnomos no se podra abrir la cerradura. Por tanto hacen falta al menos

5 144 = 720

llaves. Recprocamente si cada

llave la tienen 5 gnomos, eligiendo cualesquiera tres uno de ellos tiene una copia de esa llave por el principio del palomar.

Problema 3.

Sea

S R2 S

un subconjunto nito del plano tal que si una

recta contiene dos puntos de todos los puntos de

entonces contiene al menos 3. Demostrar que

estn alineados.

Solucin propuesta por la redaccin


Lo haremos por reduccin al absurdo. Supongamos que tenemos un conjunto de puntos

S R2

tal que, si una recta contiene dos puntos de

S,

entonces contiene al menos 3, y no todos los puntos de

son colineales.

51

Denimos una recta conectora como una recta que contiene al menos dos puntos de

S.

Sea

(P, l)

el punto y la lnea conectora separados por la mnima

distancia positiva de entre todos los pares punto-recta.

Por la suposicin, la recta conectora

S.

Lanzando una perpendicular desde

l P

contiene al menos tres puntos de a

l,

deben quedar al menos dos

puntos a un lado de la perpendicular (uno puede estar exactamente en la interseccin de la perpendicular con l). Llamemos a la perpendicular, y con

al punto de

al ms lejano. Dibujemos la recta

que conecta

l ms cercano P

C . Entonces, la distancia desde B a m es menor que la distancia desde l, contradiciendo la denicin original de (P, l). Una manera de ver esto es notar que el tringulo rectngulo con hipotenusa BC es semejante y est contenido en el tringulo rectngulo con hipotenusa P C .
a Por lo tanto, no puede haber una menor distancia positiva entre todos los

pares punto-recta, es decir, todo punto est a una distania nula de toda recta. En otras palabras, todo punto de

debe estar contenido en la misma recta si

toda recta conectora contiene al menos tres puntos.

n-simo primo. Y An (x) = xn + p1 xn1 + ... + pn1 x + pn . Demostrar que An es irreducible como polinomio sobre Z[x]. n n1 Se pide demostrar que el polinomio fn (x) = x +p1 x +. . .+pn es irreducible en Z[X] siendo pi el i-simo primo.
Problema 4.

Sea

pn

el

Solucin propuesta por la redaccin


Primero vamos a demostrar que si

fn

es reducible es

son polinomios y uno de los dos tiene trmino independiente

fn = r q , donde r 1.

52

Puesto que

Z no es un cuerpo, el primer caso que hay que descartar es que 2x + 2 = 2 (x + 1) no es fn esto no puede ocurrir

sea el producto de un polinomio por una constante (es decir, hay que probar que el contenido del polinomio sea 1). Por ejemplo irreducible sobre

Z[X].

Sin embargo, en el caso de

porque el trmino principal es

1.

en

La otra opcin que tenemos que es que fn = rq , donde r y q son polinomios x de grado al menos 1 con coecientes enteros. Si r(x) = rs xs + . . . + r0 l y q(x) = ql x + . . . + q0 , al multiplicar e igualando el trmino de mayor grado observamos que rs ql = 1, y como rs y ql son enteros, tenemos que rs = ql = 1. Multiplicando por 1 los dos polinomios, podemos suponer que los dos son 1. Ahora, igualando el trmino de menor grado tenemos que pn = r0 q0 . Como pn es un primo y r0 y q0 son enteros tenemos que uno de los dos es 1. Sin prdida de generalidad supondremos que r(x) es quien tiene trmino independiente 1. Para llegar a una contradiccin, nos damos cuenta que contenidos en

se puede ver

tambin como un polinomio con coecientes complejos, pues los enteros estn

r factoriza completamente y utilizanr(x) = (x m1 ) (x ms ). Igualando los s trminos independientes tenemos que 1 = (1) m1 ms . Lo que haremos para terminar la demostracin ser demostrar que si vemos fn como un polinomio en C todas sus races tienen mdulo estrictamente mayor que 1, y como las races de r estn entre las de fn tendramos, igualando mdulos, que 1 = |m1 | |ms | > 1, lo cual es una contradiccin. Demostrmoslo: C.
En

el polinomio

do que es mnico tenemos que

Si multiplicamos fn por (x 1) obtenemos (x 1)fn = bn+1 xn+1 + + b1 x pn , donde los bi son unos enteros mayores que 0 cuya suma es pn (ntese que 1 es raz del nuevo polinomio). n+1 + . . . + b1 z| Si tomamos un complejo |z| 1 tenemos que |bn+1 z n+1 +bn+1 |z| + . . . + b1 |z| bn+1 + . . . b1 = pn , donde hemos utilizado la desigualdad triangular. La desigualdad triangular nos dice tambin que la igualdad slo se da si los nmeros son proporcionales con una constante Tomando los dos primeros tenemos real mayor o igual que ser

b1 z = cb2 z 2 z = b1 /cb2 ,

por tanto

c 0. z es

0. Para que cumpla la segunda desigualdad tiene que |z| = 1, as que la nica solucin posible de (x1)fn para |x| 1 es x = 1. Como fn (1) > 0, todas las races de fn visto como un polinomio sobre C tienen mdulo estrictamente mayor que 1. Como las races de r son un subconjunto de stas llegamos a la desigualdad que queramos 1 = |m1 | . . . |mn | > 1 y por tanto fn es irreducible.
Observamos que el argumento se poda repetir palabra por palabra si el polinomio era de la forma

xk + bk1 xk1 + . . . + b1 x + p
53

donde

es primo y

1, bk1 , . . . , b1 , p

es una sucesin estrictamente creciente de nmeros enteros.

Problema 5.
existen

de ellos tal que su suma es divisible por

(Erds) Dados 2n 1 enteros, no necesariamente distintos, n.

Solucin propuesta por la redaccin


La demostracin la haremos en tres pasos. 1. Estamos tratando con un problema de divisibilidad, por ello es aconsejable reducirlo al caso en que para

es primo. Para eso tenemos que demostrar que la

propiedad es multiplicativa. Es decir, supuesto que es cierta para

n = a

n=b

entonces es cierta para

n = ab.
los enteros dados. Lo primero que vamos en general que

Demostrmoslo: Sean que tambin lo sea divisible por

r1 , . . . , r2ab1

a hacer es reorganizarlos de manera que

ra+1 + . . . + r2a y a para 0 k < 2b 1. a

r1 + r2 + . . . + ra sea divisible por a, rka+1 + . . . + r(k+1)a sea 2a1 primeros

Vamos a reorganizarlos as progresivamente. Podemos tomar los y de esos hay por hiptesis quedan los cuya suma es divisible por

a,

esos los aparta-

mos. Si los renombramos tenemos que

r1 + r2 + . . . + ra es divisible por a. Nos 2ab 1 a enteros restantes ra+1 , . . . , r2ab1 con los que repetimos el proceso. Como tenemos n = 2ab 1 = (2b 2) a + 2a 1, tras 2b 2 iteraciones de este proceso tenemos que las sumas r1 + . . . + ra , ra+1 + . . . + r2a , ..., r(2b3)a+1 + . . . + r(2b2)a son todas divisibles por a y nos quedan los ltimos 2a 1 de los cules conseguimos que a de ellos sean divisibles y nos olvidamos de los a 1 que nos sobran.
Ahora denimos los siguientes nmeros enteros:

y1 = y2 =
. . .

(r1 + (ra+1 + (r(m1)a+1 +

... ...
. . .

+ra )/a +r2a )/a +rma )/a +r(2b1)a )/a

ym =
. . .

...
. . .

y2b1 =
Los por

(r(2b2)a+1 + . . .

yi son en efecto nmeros enteros, pues cada una de las sumas era divisible a. Tenemos 2b1 y s y hemos supuesto que la propiedad era cierta para n = b, por lo que hay b de ellos cuya suma es divisible por b. Renombrando tenemos que b|y1 +. . .+yb . Multiplicando por a obtenemos que ab|r1 +. . .+ra +. . .+rba . Es decir, de los 2ab1 podemos elegir ab tales que su suma es divisible por ab.
54

Por tanto la propiedad es multiplicativa, si la demostramos para los primos tendremos automticamente que es cierta para todos los nmeros. 2. Como ya hemos dicho, slo nos queda demostrar el problema para los primos. Por comodidad mental llamaremos que

p a n. Una de las ventajas que tiene n = p sea primo es que Zp es un cuerpo y entonces el anillo de polinomios Zp [x1 , . . . xm ] tiene muy buenas propiedades. Para conseguir aprovecharlas Zp .

tenemos que conseguir expresar el problema que queremos resolver en trmino de polinomios en varias variables sobre Como

es primo tenemos el pequeo teorema de Fermat, que nos dice que

Zp , 1 si a = 0 y 0 si a = 0. Es importante aclarar que Zp y no la variable x de Zp [x]. De esta manera el p1 polinomio de Zp [x] dado por f (x) = x valdr 1 o 0 cuando sustituyamos por un valor, pero f = 1 en Zp [x] pues son polinomios diferentes. Una vez dicho esto ya podemos expresar el problema en Zp [x1 , . . . , x2p1 ]
vale o en esta

ap1

es un nmero de

mediante el siguiente sistema:

xp1 + . . . + xp1 1 2p1 r1 xp1 + . . . + r2p1 xp1 1 2p1


Para entender el sistema tenemos que ver los un vector

=0 =0
como contadores. Tenemos

t = (t1 , . . . , t2p1 ) que queremos ver si es solucin y cada xp1 ser i uno si la coordenada i de t es distinta de 0 y 0 en otro caso. De esta manera, en la primera ecuacin sumamos 1 por cada coordenada que es distinta de 0. Como estamos sumando 2p 1 slo hay dos posibilidades para que la primera ecuacin se cumpla, que todos las coordenadas sean 0 o que haya exactamente p coordenadas distintas de 0, pues estamos trabajando en Zp y all p = 0. En cuanto a la segunda ecuacin, sumamos los ri cuando ti es distinto de 0. Que la ecuacin sea 0 signica que la suma de los ri que corresponden a coordenadas no nulas de t sumen 0 md p. Es decir, que su suma sea divisible o por p. As, una solucin del sistema es un vector (t1 , . . . , tn ) tal que todas sus coordenadas son 0 o tal que exactamente p coordenadas son distintas de 0 y la suma de los p nmeros ri correspondientes es divisible por p. El primer caso
diremos que es la solucin trivial. Si existe una solucin no trivial entonces el teorema ser cierto. Ese es el objetivo del tercer paso de la demostracin. 3. Para terminar el problema, slo nos queda demostrar que el sistema que expresa nuestro problema tiene una solucin no trivial . Vamos a aprovechar

xp1 i

3 En teora algebraica de nmeros se conoce este resultado como teorema de ChevalleyWarning.

55

de nuevo que en

Zp se cumple el pequeo teorema de Fermat, en este caso para p, pero como al menos tenamos una, la trivial,

contar hbilmente las soluciones del sistema. Demostraremos que la cantidad de soluciones es divisible entre habr tambin una no trivial. Para ello, llamamos

f1 = xp1 + . . . + xp1 y f2 = r1 xp1 + . . . + 1 2p1 1 p1 r2p1 x2p1 , los dos polinomios que nos daban el sistema. Denimos f := p1 p1 (1 f1 )(1 f2 ). f es un polinomio en las variables x1 , . . . , x2p1 y si t es un vector de 2p1
coordenadas con valores en distinto de

Zp

se cumple que

f (t) = 1

si y slo si

es una

solucin del sistema. En efecto, si

no es solucin del sistema

f1 (t)

f2 (t)

es

y al elevarlo a

p1

nos da

1,

que cancela uno de los factores.

Recprocamente, si donde

es solucin del sistema,

La manera de contar las soluciones, mdulo

f (t) = 1. p, ser

por tanto sumar

f (t)
con

recorre los vectores de

2p 1

coordenadas. Es decir

N=

f (t)

t Z2p1 . p

N es cero. e2p1 e1 El polinomio en 2p1 variables f (t) ser una suma de monomios x1 x2p1 . Se cumple que en cada monomio uno de los ei es menor estricto que p1. Si no fuera as, f (t) tendra un monomio con grado e1 +. . .+e2p1 (2p1)(p1). p1 p1 Sin embargo, f estaba denido por (1f1 )(1f2 ), los dos factores tienen 2 grado (p 1) por tanto f (t) tiene grado (2p 2)(p 1) < (2p 1)(p 1), as que f (t) no puede tener un monomio de grado (2p 1)(p 1) y por tanto cada monomio tiene al menos uno de los xi elevado a un ei que es menor estricto que p 1.
Como habamos anunciado tenemos que demostrar que Esta acotacin al grado de uno de las variables nos sirve por la siguiente

n rZp r = 0. Antes de demostrarla comprobemos que nos sirve para demostrar lo que queremos. Por comodidad
propiedad de

Zp :

si

n < p1

entonces

en la notacin, supondremos que en un cierto monomio elevado a un nmero menor que

x1

es el que est

p 1.
2p1 te2 t2p1 2

2p1 te1 t2p1 = 1

t e1 = 0 1
t1 Zp

t1 ,...,t2p1

t2 ,...,t2p1

donde simplemente hemos aplicado la propiedad distributiva y la propiedad de

Zp

que ahora demostraremos. Por tanto, la suma de un monomio de

f (t)

evaluado en todos los vectores es propiedad.

0.

Por denicin

N =

f (t),

sumando

cada monomio por separado tenemos que

N = 0.

Slo queda demostrar la

Hemos recordado antes que todo elemento no nulo de Se puede anar ms, existe un nmero cuyo orden es

Zp elevado a p 1 era 1. p 14 . Es decir ap1 = 1

4 ste es un resultado estndar que se demuestra en teora elemental de nmeros, as que no reproduciremos la demostracin aqu

56

ar = 1

a
es

una raz primitiva,

r < p 1. A un nmero as se le conoce como raz primitiva. Sea a tiene inverso as que se cumple que a 1, a 2, . . . , a p 1, 2, . . . , p en otro orden. Entonces:
si

tr =
tZp tZp

(at)r

(a 1)
tZp

t =0 tr = 0
tZp

donde hemos utilizado que

r <p1

y por tanto

ar 1 = 0.

De esta manera

queda demostrada la propiedad.

Resumiendo, en cuerpo es que

Zp

la suma de

xr

cuando

r < p1

recorre todo el

0.

Como

f (t)

era un polinomio en varias variables que era suma la suma de

de monomios cada uno con el grado de una de sus variables menor estricto

p 1, tenamos que 2p 1 coordenadas era 0.

Esta suma era

de nuestro sistema mdulo implicaba que haba para todo

p.

Como

f (t) cuando t recorra los vectores de N , que era la cantidad de soluciones t = (0, . . . , 0) era solucin del sistema, p.

al menos hay una solucin no trivial. Una solucin no trivial del sistema nos

de los nmeros dados cuya suma era divisible por

Como tenamos demostrado que sta propiedad era multiplicativa, es cierta

n.

Problema 6.
tal que

Problema abierto.

(Ulam) Existe un subconjunto donde

de

R2

es denso y

p, q S , d(p, q) Q,

es la mtrica usual?

Comentarios de la redaccin
Aunque el problema sigue abierto, se puede generalizar de varias maneras que s han sido resueltas. En el 2008 Jozsef Solymosi y Frank de Zeeuw

5 probaron que si una curva

algebraica plana contiene innitos puntos de manera que la distancia entre dos cualesquiera es racional, entonces la curva es un crculo o una recta. Si en lugar de distancias racionales nos interesan distancias enteras, el teorema de ErdsAnning nos dice que si un conjunto de innitos puntos

5 http://arxiv.org/abs/0806.3095

57

cumple que cada dos estn a distancia entera entones los puntos estn en una recta. Una generalizacin alternativa es llevarlo a espacios topolgicos. Aunque no es cierto que cualquier espacio mtrico completo y separable junto denso de manera que la distancia entre dos puntos suyos sea racional, s es cierto que existe una mtrica equivalente, ie que dene la misma topologa, de manera que hay un subconjunto denso con la propiedad requerida. Por ejemplo, en el plano esa mtrica sera la mtrica de Manhattan y el conjunto denso

6 tenga un con-

Q Q.

La demostracin de estos dos resultados, referencias a las

generalizaciones ya mencionadas y otros resultados se pueden encontrar en

http://mathoverflow.net/questions/19127/is-there-a-dense-subsetof-the-real-plane-with-all-pairwise-distances-rational .

6 Con un subconjunto numerable denso.

58

Curiosidades
La curiosa mquina del doctor Zavrotsky

El Doctor Andrs Zavrotsky


El pasado verano tuve la suerte de asistir a una conferencia del profesor mexicano Carlos Bosch, dentro del curso VI Escuela de Educacin Matemtica Miguel de Guzmn, que organizaba la RSME junto con la Universidad de Burgos. El tema all tratado era el de la enseanza de las matemticas desde primaria a la universidad, y Carlos Bosch nos puso un ejemplo prctico de lo que se puede considerar una motivadora e interesante clase de matemticas. La conferencia se articulaba en torno a su libro de divulgacin El billar no es de vagos [1], en el que explica diversas experiencias que relacionan el mundo del billar con las matemticas. En una de sus partes, el autor explica el funcionamiento de una curiosa mquina de luz y espejos, la mquina de Zavrotsky. Antes de explicar su funcionamiento, contar algo de la interesante vida de su creador.

Andrs Zavrotsky naci en San Petersburgo en 1904 y muri en la ciudad de Mrida (Venezuela) el 26 de diciembre de 1995. Ejerci la docencia en la Universidad Central de Venezuela desde 1947 hasta 1952 y en la Universidad de Los Andes (Venezuela) desde 1952 hasta su jubilacin en 1974. Poco se sabe de la vida de Zavrotsky antes de su llegada a Venezuela a nales de los aos 40, salvo que fue el admirador del lsofo ruso Losky, del que fue discpulo en San Petesburgo, y del cual tradujo la obra La intuicin sensorial, intelectual y mstica al castellano. Hebertt Sira [3], nos cuenta las circunstancias en las que lo conoci, alrededor del ao 50, siendo su alumno de anlisis matemtico en el primer curso de la Escuela de Forestal. Segn cuenta Sira, Zavrotsky era concienzudo en su trabajo e impecable en sus explicaciones, y gozaba de una asombrosa habilidad para el clculo mental (se deca de l que poda calcular el logaritmo del nmero de la matrcula de cualquier coche mientras se le acercaba). A parte de su labor docente e investigadora en la universidad, Zavrotsky destac por su continua actividad en multitud de disciplinas. En 1952, Andrs Zavrotsky y Fausto Gonzlez realizaron, como tutores de los hoy ingenieros Alejandro Rivero y Ernesto Pacheco, una pelcula en dibujos animados, 16 mm, blanco y negro, 3 minutos de duracin, sobre el modelo matemtico

Tesaracto o Hipercubo con la nalidad de facilitar la visualizacin de

la cuarta dimensin. Esta pelcula fue ofrecida para su exhibicin durante el desarrollo del Congreso Internacional de Matemticas que se celebr en Edim-

59

burgo en 1958, como testimonia la resea aparecida en el American Mathematical Monthly, Vol LXV, N. 6, Junio-Julio, 1958. En otra ocasin, segn cuenta Sira, se comentaba por Mrida que el Profesor Zavrotsky haba calculado algn tramo, quizs el ltimo, del para entonces recientemente inaugurado Telefrico de Mrida. Su contribucin habra sido necesaria por cuanto, ante la dicultad del problema tcnico, la compaa francesa que estaba construyndolo quera evadir responsabilidades por no encontrar solucin prctica para la construccin de ese tramo, el cual es, ciertamente, uno de los ms largos y el ms peligroso de todo el recorrido. Habra sido el Dr. Zavrotsky quien les demostrase a los ingenieros franceses la factibilidad de la construccin en base a clculos irrefutables. Todava hoy, el telefrico de Mrida es el ms alto del mundo. Zavrotsky se interes intensamente por otras actividades como la losofa, la literatura, la lingstica (hablaba perfectamente ruso, castellano, ingls y francs, y al parecer se defenda tambin en alemn) o la biologa. Siempre fue esquivo de las entrevistas, fotografas, homenajes y otros reconocimientos. Se conserva la conferencia que dio en 1994 en la Academia de Mrida y que est publicada en el boletn de dicha academia de 1996. Tras su muerte, su biblioteca personal fue donada a peticin suya a la Universidad de los Andes. Como dice Oswaldo Araujo [2], abrigamos la esperanza de que pronto la obra de Zavrotsky ser editada, sus libros estarn en las bibliotecas de la ULA y sus inventos en el Museo de Ciencia y Tecnologa de Mrida. De esa forma, nios, jvenes y adultos tendremos la magnca oportunidad de convivir con ese extraordinario ser que fue el Profesor Andrs Zavrotsky.

ptica y mximo comn divisor


Zavrotsky invent un aparato ptico que calcula el mximo comn divisor, cuya patente registr en abril de 1961 (U.S. Patent 2978816). La mquina sirve para calcular el mximo comn divisor de dos nmeros enteros positivos

b,

con

a = b. ab
con

Para explicar su funcionamiento, consideremos un rectngulo de dimensiones lado horizontal sea coordenadas

b > a, y pongmoslo de forma que el lado vertical sea a, el b y el vrtice inferior izquierdo se encuentre en el origen de

(Ver gura 8, en la que se pone un ejemplo de un rectngulo

de dimensiones

5 8). O
con un ngulo de 45

Ahora lancemos un rayo de luz desde el origen

grados. Supongamos que los lados del rectngulo son espejos, de forma que un rayo de luz se reeja especularmente en los lados hasta llegar a uno de los vrtices del rectngulo.

60

Figura 8: Rectngulo de

5 8.

Veremos que el rayo, tras un nmero nito de rebotes, siempre llega a un vrtice del rectngulo, que adems es distinto del origen.

En ese momento, en el eje X habr un punto B iluminado que sea el ms cercano al origen. Pues bien, la mitad de la distancia de ese punto al origen resulta ser el mximo comn divisor de a y b. Las pruebas.
Veamos primero que si de los otros tres vrtices. Como el rectngulo tiene medidas a por b, con b > a, tomemos la fraccin b/a y sea p/q la fraccin reducida igual a b/a, es decir, p/q = b/a, de donde p a = q b. Sea un cuadrado de lados bq y ap. Desde luego que ste es el ms pequeo que se puede formar con rectngulos de lados b y a. Por lo tanto, hay q rectngulos de lado b en el eje X y p rectngulos de lado a en el eje Y . Si el rayo empieza en el origen, con un ngulo de 45 grados, su trayectoria ser la diagonal del cuadrado. Si desdoblamos la trayectoria del rayo, sta llegar al vrtice superior derecho del cuadrado que acabamos de construir. Al reconstruir la trayectoria tendremos que terminar en un vrtice del rectngulo. Por tanto, al cabo de un nmero nito de rebotes el rayo llega a un vrtice del rectngulo, donde detenemos la trayectoria. Como es necesario localizar la mnima distancia entre el origen y la trayectoria sobre el eje

son enteros, el rayo que parte del origen con

un ngulo de 45 grados, siempre llega, tras un nmero nito de rebotes, a uno

X , es importante hacer notar que la trayectoria no regresar

al origen. Razonemos como sigue para ver esto. Supongamos que que la trayec-

61

Figura 9: Plano de la mquina construida por Zavrotsky [4].

toria vuelve al origen. Entonces, para regresar debe seguir la trayectoria de salida, pues al haber salido con un ngulo de 45 grados no hay otra posibilidad, lo cual quiere decir que el rayo va y regresa por el mismo camino sin pasar antes por un vrtice del rectngulo. Esto es imposible, ya que el punto de regreso debe estar sobre una banda o dentro del rectngulo y no ser un vrtice. Si el rayo est dentro, seguir su trayectoria en lnea recta y no regresar al origen. Si por el contrario, el punto est en una banda rebotar con un ngulo de reexin igual al de incidencia, es decir, de 45 grados, por lo que tampoco puede regresar por el camino de llegada. Se puede hacer un estudio de a qu vrtice de los tres restantes llegar el rayo, haciendo uso de la paridad de interesa. Vamos con la demostracin formal de que la distancia el doble del mximo comn divisor de

(ver [1] y [4]) pero aqu no nos

OB

es, en efecto,

b.

Observemos la gura 3, que

representa el rectngulo desdoblado hasta formar un cuadrado de lado

ap=

p b a = q y la ltima fraccin es irreducible. As, nuestro rayo describe una recta, la diagonal del cuadrado.

b q , donde

62

Figura 10: El rayo no regresa al origen.

Trabajaremos en el anillo

Z2b .

Aqu, las coordenadas de los

Oi ,

es decir,

los puntos que va ocupando el origen

en el cuadrado, son

Oi = (0, 2ai),
mientras que las coordenadas de los

i = 1, . . .
son

p , 2 p , 2 b,
viene dada

Bi

Bi = (2ai, 2ai), ,
donde por

i = 1, . . .

denota la parte entera de

x.
que siempre es menor que si si

As, la distancia entre

Oi

Bi ,

d(Oi , Bi ) =

2b 2ai, 2ai,

2ai > b; 2ai b.

Recordemos ahora el conocido resultado de teora de nmeros

2 mcd(a, b) = mn{2am + 2bn,


Entonces, en

m, n Z,

2am + 2bn > 0}

Z2b

se tiene que

2 mcd(a, b) = 2d = mn{2am

(md 2b) : o

m Z,

2am = 0

(md 2b)}. o

Por tanto, slo falta demostrar que min{d(Oi , Bi ),

i = 1, . . . ,

p } = 2d. 2

Para verlo, consideremos la congruencia

2aj 2d (md 2b) o


63

Figura 11: Cuadrado de lado

p a = q b.

Como

2d|mcd(2a, 2b)

(de hecho se cumple la igualdad), la congruencia

tiene solucin, y adems existen

2d soluciones en Z2b . Adems, se cumple que 2ab 2a d (md 2b), o

2a( + p) = 2a + 2ap = 2a +
por lo que podemos asegurar que existe la congruencia (Si

j = p,

entonces

m {1, . . . , p 1} que es solucin de 2aj 0 (md 2b), luego j no es solucin). o 2d b.


Pero los puntos

Por otra parte, como

b > a,

entonces

{2aj :
de

j = 1, . . . , p 1 ;

2aj Z2b } {0, 2b} = S {0, 2b} [0, 2b], es decir, que slo

estn equiespaciados en el intervalo

p 2 de los puntos son menores o iguales que b. Por ltimo, notemos que los puntos d(Oi , Bi )

son todos de la forma o iguales que e

2aj

en

Z2b , con j {1, . . . p 1} y adems son menores d(Oi , Bi ) = d(Oj , Bj ),


entonces con

b.

Por tanto, si probamos que todos son distintos habremos ter-

minado. Pero, supongamos que

i, j {1, . . . ,

p 2

i = j.
Si

Entonces distingamos tres casos:

d(Oi , Bi ) = 2ai

d(Oj , Bj ) = 2aj ,
y

2ai 2aj i = j
entonces

Contradiccin. Si d(Oi , Bi ) = 2b 2ai i = j Contradiccin.

d(Oj , Bj ) = 2b 2aj ,

2ai 2aj

64

Por ltimo, si

d(Oi , Bi ) = 2ai y d(Oj , Bj ) = 2b 2aj , 2aj i + j 0 Contradiccin.

entonces

2ai

Referencias
[1] Carlos Bosch Giral, El billar no es de vagos. Ciencia, juego y diver-

sin,
Mxico: FCE, SEP, CONACyT, 2009. [2] Oswaldo Araujo G, La ejemplar vida de Andrs Zavrotsky, Boletn de la Asociacin Matemtica Venezolana Vol. IV, No. 2 (1997). [3] Hebertt Sira Ramrez, El Dr Andrs Zavrotsky que an puedo recordar, Boletn de la Asociacin Matemtica Venezolana Vol. IV, No. 2 (1997). [4] Andrs Zavrotsky, Greatest common divisor nder, United States Patent Oce, 2978816 Apr. 11, 1961.

http://www.freepatentsonline.com/2978816.pdf
Vctor Arnaiz

Carreras de primos

Puesto que todos los primos excepto el 2 son impares, podemos dividirlos en dos conjuntos: el conjunto 4, y el

C1 ,

formado por los congruentes con 1 mdulo

C2 ,

en el que aparecern los congruentes con 3 mdulo 4, y pregun-

tarnos como se van distribuyendo entre estos dos conjuntos. Para hacernos una primera idea de lo que ocurre clasicaremos los nmeros menores que una cota

n0 .

En la tabla 1 se considera 5 13 7 11 53 61 59 67 17

n0 = 100.
29 37 31 73 71 y 79 con 83 41

C1 C2

19

23


89 97

43

47

Tabla 1: Los conjuntos

C1

C2

n0 = 100.

65

En ella observamos que ambos conjuntos tienen un tamao parecido y tambin que, aunque cambiemos la cota para diferentes valores de

n0

a otra menor,

nunca tiene ms

elementos que B. En la tabla 2 se recojen los cardinales de los dos conjuntos

n0 . n0
100 500 1000 5000 10000

|C1 |
11 44 80 329 609

|C2 |
13 50 87 339 619

Tabla 2: Cardianles de

C1

C2 .

Este hecho no es exclusivo para las cotas que se muestran. Cualquier cota que hubisemos elegido,

n0 < 10000, habra arrojado el mismo resultado: |C1 | |C2 |, como puede comprobarse haciendo un simple programa informti-

co. Lo cierto es que fue Tchebyshev quien primero se percat de este hecho; y aunque probablemente sus cotas no llegasen a ser tan grandes como stas, se conjetur que ocurrira as siempre. Si esta conjetura hubiese resultado cierta desde luego que tendramos para una buena curisidad, pero la realidad es an ms sorprendente. Como he

C1 tiene C2 , aunque como 26863 tambin es primo las cosas quedan igualadas, y los siguientes nmeros hacen que C2 siga siendo ms grande que C1 . Sera ste un hecho puntual o volvera a quedar C1 por delante? De nuevo
anticipado, la conjetura no se cumple, y es que con el primo 26861, un primo ms que podemos comprobar que las cosas cambian con la llegada del primo 616841 que hace que

C1

tenga ms primos que

C2 , aunque de nuevo esta superioridad

desaparece tras unos cuantos nmeros ms. Llegados a este punto puede parecer que la cosa a perdido su gracia, la conjetura que se hizo Tchebychev ha resultado ser falsa ya que hemos encontrado algn contraejemplo, pero lo mejor est por llegar con la aparicin de Littlewood, quien sentenci la cuestin demostrando el siguiente teorema:

Teorema 1. (Littlewood, 1914) Existen valores


tales que

n0

arbitrariamente grandes

|C1 | > |C2 |.

De hecho, se tiene que

|C1 | |C2 |

1 x ln ln ln x, 2 ln x
66

con lo que podemos encontrar una cota en la que ms que

C1

tenga tantos elementos

C2

como queramos.

No obstante, aunque el teorema enunciado por Littlewood muestra que si elegimos un

n0

cualquiera podremos encontrarnos con que

|C1 | > |C2 |,

Cul es la probabilidad de que esto ocurra? Esto fue lo que se preguntaron Knapowski y Turn. Ms concretamente, su pregunta fue si la probabilidad de que

|C2 |

sea mayor que

|C1 |

al elegir un cierto

n0

tiende a 1 cuando

n0

tiende a innito. Es decir,

1 = l m

1 #{n0 < n : n

|C2 | > |C1 |}

Una vez ms la conjetura result falsa, y la falsedad de la conjetura dio lugar a una respuesta ms apasionante. Esta vez fueron Kaczorowski y Sarnak los encargados de demostrar que

1 #{n0 < n : n

|C2 | > |C1 |}

no slo no tena lmite 1, sino que no tena lmite. Poco ms tarde Rubinstein y Sarnak se percataron de que haba otra forma de contar los tamaos de ambos conjuntos con la que obtener una respuesta ms satisfactoria. Lo que hicieron fue en lugar de sumar 1 por cada por cada

n0

en que

|C2 | > |C1 |. 1 = l m

Por tanto,

|C2 | > |C1 |, sumar 1 recordando que ln x = x,


en que

1 x la

reformulacin de la conjetura sera

1 ln n

n0 <n: |C2 |>|C1 |

1 , n

lo que tampoco ocurre, pero casi, ya que el lmite antes descrito es aproximadamente 0.9959, es decir, que en el 99.59 por ciento de las ocasiones en las que elijamos Tchebychev.

n0

estaremos ante la situacin que observ en un primer instante

Referencias
[1] Andrew Granville y Greg Martin, Carreras de nmeros primos, La Gaceta de la RSME, Vol. 8.1 (2005), Pgs 197-240

Pablo Aguado

67

Esferas exticas
Una de las aportaciones ms importantes que John Milnor ha dado a las matemticas es la apertura de un nuevo camino de investigacin en ellas: la topologa diferencial. En esta curiosidad veremos cmo y por qu se abri esta nueva lnea de investigacin analizando las esferas exticas. El primer concepto que deberemos manejar es el de variedad. Para aproximarnos a este concepto pensemos por ejemplo en la supercie de una esfera. Si cortamos un trozo muy pequeo de esta supercie, se parecer a un trozo plano de papel. Es como nuestra estancia en la Tierra, que desde nuestro punto de vista nos parece plana, pero si nos alejamos los suciente observaremos que es muy distinta. Como un papel es dimensional, consideramos la supercie de la esfera tambin bidimensional. Ntese que si solo consideramos este trozo, su geometra parecer eucldea, aunque el comportamiento global de nuestra variedad no ser como el de la geometra plana; en nuestro caso, tendremos una geometra esfrica. Cuando pensamos en una variedad como la supercie de una esfera, la visualizamos en un espacio de 3 dimensiones, pero podemos hablar de las variedades de forma intrnseca, rerindonos solamente a los puntos de nuestra variedad. Adems, hay variedades de cualquier dimensin. Aunque no podamos visualizar muy bien una variedad de 7 dimensiones, podemos estudiar sus propiedades de forma abstracta. Sin embargo no hemos dicho hasta qu punto se tiene que parecer una variedad a otro objeto. De hecho, hay muchas formas de considerar que dos objetos se parezcan, aunque para nuestro caso analizaremos dos nociones. La primera nocin es topolgica: diremos que dos formas son homeomorfas si una es una deformacin continua de la otra. Por ejemplo, un tringulo es homeomorfo a una circunferencia, y una taza a un toro.

Si nuestra forma es tal que trozos sucientemente pequeos son homeomorfos a trozos de espacio eucldeo, diremos que esa forma es una variedad topolgica. La segunda nocin va a ser ms fuerte: si adems de exigirle a nuestra deformacin que lleva de una forma a otra que sea continua le obligamos a

68

que sea diferenciable, entonces nuestras formas son difeomorfa, ya que en el cubo tenemos picos.

difeomorfas. Ntese que

la supercie de un cubo es homeomorfa a la supercie de una esfera, pero no

Si con las deformaciones continuas conseguamos que no se rompiese nuestra forma, con las deformaciones diferenciables conseguimos evitar que aparezcan picos o esquinas, ya que son deformaciones ms suaves. As, si seguimos un camino diferenciable en la forma inicial, al transformarse el camino tambin ser diferenciable. Ahora es cuando viene lo bueno: en 1956, Milnor encontr un objeto matemtico extraordinariamente curioso: se trata de una forma homeomorfa a una esfera de 7 dimensiones pero no difeomorfa a la esfera. En principio esto no nos llama demasiado la atencin, pues podemos pensar en un hipercubo de 7 dimensiones (que cumple tambin la propiedad), pero la sorpresa aparece cuando observamos que el objeto de Milnor es diferenciable, suave, sin picos ni aristas. Ntese que un hipercubo tendr picos y esquinas, por lo que parece contraintuitivo que un objeto as pueda existir. Milnos llam a este objeto esfera extica. Analicemos con ms detalle por qu esto parece contraintuitivo: si cogemos una esfera y la deformamos de forma continua, sin romperla, y llegamos a una nuestra esfera extica, que es una supercie suave, las nicas deformaciones que llevan de una a otra

no son suaves.

Uno puede verse tentado a aplanar los picos de la deformacin para obtener una deformacin diferenciable entre ambas formas. Por ejemplo, si tenemos una circunferencia, una forma suave homeomorfa a la circunferencia ser una curva cerrada suave, es decir, una curva que empieza y acaba en el mismo punto, sin picos y que no se atraviesa a s misma. Se puede demostrar que estas formas son difeomorfas, es decir, que existen deformaciones diferenciables que llevan la circunferencia a la curva en cuestin. Es tentador pensar que podemos hacer esto en cualquier dimensin: si nos dan una variedad suave homeomorfa a una esfera, intentaremos modicar la deformacin en los sitios en los que no es suave para conseguir una deformacin suave. Y por qu esto no funciona? Una de las razones es que no es posible aplanar la deformacin globalmente. Si pensamos en una supercie de 5 dimensiones, los sitios en los que la deformacin no es suave puede ser un conjunto tridimensional raro. As, al aplanar por un sitio, puede que desplaces tu pico a otro sitio. Si tienes suerte, puedes conseguir que la arruga que en un sitio sube se cancele con otra arruga que baja, pero el ejemplo de Milnor nos demuestra que esto no ocurre siempre. Pero cmo son estas esferas exticas? Veamos que no son difciles de

69

denir. Al igual que una esfera est compuesta de dos partes topolgicamente equivalentes a discos (los hemisferios), una esfera de 7 dimensiones tambin la podemos formar con dos discos de 7 dimensiones. Estos discos se unen en un ecuador de 6 dimensiones. En realidad, hay dos copias del ecuador, una en el borde del hemisferio norte y otra en el borde del hemisferio sur. Podemos unir ambos hemisferios pegando cada punto del ecuador norte al mismo punto del ecuador sur. Pero tambin podemos conseguir unir los puntos del ecuador del sur a puntos diferentes del ecuador norte, retorciendo y aplastando el ecuador de forma que conseguimos una variedad homeomorfa a la esfera pero no difeomorfa. El hecho de que variedades pudiesen ser homeomorfas y no difeomorfas signicaba que las variedades diferenciales eran objetos matemticos importantes, por lo que esta construccin supuso el nacimiento de la topologa diferencial como campo de las matemticas con derecho propio. Esta es una aportacin ms del genial John Milnor. El premio Abel que le ha sido otorgado es bienvenido por matemticos de todo el mundo.
Basado en el artculo

The work of John Milnor, de Tim Gowers Pedro ngel Castillejo

70

Retos y pasatiempos
Criptografa
Seguimos con la serie de problemas criptogrcos propuestos por Wirieg. Habr un premio para el primer lector que resuleva todos los problemas (esto incluye los ya propuestos en el primer nmero de Matgazine, que si no tenis podis ver en nuestra pgina web, y los problemas que aparecern en el siguiente nmero).

Problema 3. Qu letra continuara esta serie?


M P T Z J Y

Problema 4. En qu sentido gira naturalmente el agua en nuestro hemisferio? Para "positivo", 3; para "negativo", 2. Anota aqu los resultados de los ejercicios, y no los olvides! (Las dos primeras casillas corresponden a los problemas 1 y 2 del primer nmero).

(1)

(2)

(3)

(4)

Retos de piscina, playa o montaa

1. Pueden recorrerse estos nueve puntos con una lnea continua formada por cuatro segmentos?

71

2. Cmo habrn de plantarse 4 rboles para que entre ellos halla la misma distancia? 3. Una bacteria se reproduce dividindose en dos una vez por minuto. Si dos bacterias son capaces de reproducirse hasta llenar un tubo de ensayo en 4 horas y 38 minutos, cunto tardar una sola bacteria en lograr lo mismo? 4. Cambiando un slo dgito de posicin, qu igualdad se puede obtener a partir de la siguiente expresin?

62 63 = 1
5. Colquense los nmeros del 1 al 8 en estas casillas evitando que sean consecutivos, lateral o diagonalmente. Nota: Hay ms de una solucin.

6. Encuntrense los valores de las letras para que se cumpla la igualdad (cada letra representa un dgito y a letras distintas les corresponden valores distintos):

(M I)2 = CASA
7. Pueden colocarse los nmeros del 1 al 8 en los cuadrados con la condicin de que la diferencia entre dos nmeros vecinos no sea nunca menor que 4?

72

8. Colquense en los vrtices de este cubo los nmeros del 0 al 7 de manera que la suma de los dos nmeros de cada arista sea un nmero primo.

9.

Puede formarse un cubo con 5 cerillas (segmentos)?

Una curiosidad.

Cul es el nmero que multiplicado por 49 resulta un nmero que se escribe slo con la cifra 1? Resulta que el nmero es el 2.267.573.696.145.124.716.553.287.981.859.410.430.839 No probis a comprobarlo con una calculadora comn, ni siquiera con las versiones de serie de Windows XP, Vista o 7, porque en ninguna de ellas cabe (aunque podis obtener una aproximacin metiendo el nmero incompleto). Sin embargo, en la calculadora estndar de Ubuntu lo podis ver en un momento.
Nmeros Mgicos. 495

Escjase un nmero cualquiera de tres cifras, no todas iguales; por ejemplo, el 527. Constryase otro ordenando las cifras de mayor a menor: 752. Despus ordenndolas de menor a mayor: 257. Rstense al mayor el menor: 752 257 = 495. Si no resultara a la primera basta repetir el proceso: Ejemplo: 474. 744 447 = 297. 972 279 = 693. 963 369 = 594. 954 459 = 495. Lo mismo pasa con el nmero 9 y los nmeros de dos cifras y el 6714 para los nmeros de cuatro cifras.

73

124

Este nmero tiene la propiedad de que cualquier separacin de sus dgitos da como resultado dos nmeros, uno mltiplo del otro siempre. 1 y 24; 12 y 4. El mayor nmero que cumple esto es el 3162: 3 54 = 162; 31 2 = 62; 316 = 2 158. Si se considera el 0, el nmero es el 31806: 3180/6 = 530; 318/6 = 53; 806/31 = 26; 1806/3 = 602.
481

Dado un nmero de dos cifras cualquiera, ab, constryase el nmero ab+ab20. Sea cde ese nmero. Multiplquese cde por 481. El resultado siempre es ababab, siendo ab el nmero escogido al principio.

Ms retos

10.

Es posible dividir esta gura en cuatro piezas idnticas?

11. Dados estos cinco cuadrados formados por 16 segmentos, se pueden formar cuatro cuadrados idnticos moviendo solo dos palillos?

74

12. Cul es el menor nmero que, dividido por 2, 3, 4, 5 y 6 da respectivamente los restos 1, 2, 3, 4 y 5?

13. Se pueden expresar todos los nmeros del 1 al 10 usando slo cuatro treses y las operaciones +, -, , /, !, y las potencias?

14. Una hormiga llega a un tablero 5 5 como el de la imagen, a la posicin sealada. Si a la hora de empezar su paseo decide ir de una casilla a otra solo horizontal y verticalmente, sin repetir ninguna casilla en su trayecto, podr recorrer las 25 casillas?

Miguel Ardura

75

Soluciones a los pasatiempos del nmero 0.

Crucigrama

Retos

1. Te ves capaz de cortar esta cruz simtrica en cinco piezas, de modo que una de las piezas sea una cruz menor simtrica y las otras cuatro restantes encajen entre s formando un cuadrado perfecto?
Solucin:

2. Con los nmeros 1,1,2,3,3,4,4,5,7, puedes hacer una fraccin que sea igual a 1/11?
Solucin:

4321/47531
76

Você também pode gostar